Categories
Chicago Economists Money and Banking

Chicago. Ph.D. Thesis Committees in Monetary Economics. Patinkin’s Research, 1968

The first boxes of archival material that I examined as my research project on the evolution of graduate economics training was beginning to take shape came from Don Patinkin’s papers back when Duke’s Economists’ Papers Archive still bore the modest descriptor of “Economists’ Papers Project”.

This post transcribes some of the research material collected by Patinkin in his survey of Chicago style monetary economics. Fun Fact: his research assistant while on leave at M.I.T. was the graduate student Stanley Fischer, from whom incidentally I was to take my first graduate macroeconomics course (Patinkin’s book was on the reading list, surprise, surprise).

Doctoral theses advisers were identified for a dozen and a half Chicago theses that drew Don Patinkin’s attention. This is the sort of information that doesn’t normally jump at you in digitised form through a duly diligent internet search, so I thought it worth my time to file this information for now in a blog post. Minor additions have been added in square brackets for the sake of completeness.

______________________________

List of Patinkin’s copy request for Chicago Ph.D. theses

Author

Article Details of parts photographed

Box No.

1.
Bach, George [Leland]

Price Level Stabilization: [Some Theoretical and Practical Considerations]

[blank]

[blank]

2.
Bloomfield, Arthur [Irving]

International Capital Movement and the American Balance of Payments 1929-1940 Title, Contents, Bibliography.
pp. 513-514, 578-579.

T-304

3.
Bronfenbrenner, Martin

Monetary Theory and General Equilibrium Title, Preface, Bibliography.
Chaps. 1, 4, 7, 8, 9, 10, 11.

T-10250

4.
Brooks, Benjamin [Franklin]

A History of Monetary Theory in the United States Before 1860 Contents, Preface, Bibliography.
Chap. 11.

T-9885

5.
Caplan, Benjamin

The Wicksellian School—A Critical Study of the Development of Swedish Monetary Theory, 1898-1932 Title, Contents, Preface, Bibliography.

T-7847

6.
Cox, Garfield V.

Business Forecasting in the United States 1919-1928 Title, Contents, Preface, Bibliography.

T-17-91

7.
Daugherty Marion [Roberts]

The Currency-Banking Controversy Title, Contents, Bibliography
pp. 41, 54, 130, 133, 246, 316.

T-10282

8.
Harper, [William Canaday] Joel

Scrip and Other Forms of Local Money Title, Contents, Bibliography.

T-145

9.
Leigh, Arthur Hertel

Studies in the Theory of Capital and Interest Before 1870 Title, Contents, Bibliography.

T-554

10.
Linville, Francis [Aron]

Central Bank Co-operation Title, Contents, Bibliography.

T-11508

11.
McEvoy, Raymond H.

The Effects of Federal Reserve Operations 1929-1936 Title, Contents, Preface Bibliography.

T-7731

12.
McIvor R. Craig

Monetary Expansion in Canadian War Finance, 1939-1946 Title, Contents, Bibliography.

T-10268

13.
McKean, Roland Neely

Fluctuations in Our Private Claim-Debt Structure and Monetary Policy Title, Contents, Bibliography.
Chaps. 1, 2, 3, 4, 5, 6, 7, 8

T-90

14.
Reeve, Joseph [Edwin]

Monetary Proposals for Curing the Depression in the United States 1929-1935 [blank]

T-11022

15.
Shaw, Ernest Ray

The Investment and Secondary Reserve Policy of Commercial Banks Title, Contents, Preface, Bibliography.

T-8322

16.
Snider, Delbert [Arthur]

Monetary, Exchange, and Trade Problems in Postwar Greece Title, Contents, Bibliography.

T-1031

17.
Tongue, William [Walter]

Money, Capital, and the Business Cycle Title, Contents, Preface, Bibliography.

T-670

Source: Duke University. David M. Rubenstein Rare Book & Manuscript Library, Economists’ Papers Archive. Don Patinkin Papers, University of Chicago School of Economics Raw Materials, Box 2, Folder “Chicago, general (?). from binder: “U. Chicago Ph.D. Theses”, folder 1 of 2”.

______________________________

The University of Chicago
Chicago, Illinois 60637

Department of Economics

August 21, 1968

Professor Don E. Patinkin
Economics Department
Massachusetts Institute of Technology
Cambridge, Massachusetts

Dear Professor Patinkin:

            I am listing below the information (Committee members) you requested in your letter of July 8, 1968. I am also hoping that you have received your microfilm by now. The Photoduplication department was to have mailed them to you on August 13.

Bach, George [Leland] 1940 S. E. Leland
C. W. Wright
H. C. Simon
Bloomfield, Arthur [Irving] 1942 J. Viner
Lloyd W. Mints
O. Lange
Bronfenbrenner, Martin 1939 Frank Knight, chr.
S. E. Leland
Brooks, Benjamin [Franklin] 1939 Frank Knight, chr.
Lloyd Mints
[Viner also thanked in thesis preface]
Caplan, Benjamin 1942 J. Viner
O. Lange
L. W. Mints
H. C. Simons
Cox, Garfield [V.] 1929 Lionel D. Edie, chr.
Jacob Viner
Chester W. Wright
Daugherty, Marion [Roberts] (Mrs.) 1941 Jacob Viner, chr.
Garfield Cox
Lloyd Mints
Harper, Joel [William Canady] 1949
[Summer 1948]
F. Knight
O. Lange
H. Simons
C. W. Wright
L. Mints
S. Leland
Leigh, Arthur [Hertel] 1946 Frank Knight, chr.
Jacob Viner
Oskar Lange
McEvoy, Raymond [H.] 1950 Lloyd W. Mints, chr.
Earl J. Hamilton
Lloyd A. Metzler
McIvor, Russel [Craig] 1947 Roy Blough, chr.
J. K. Langum
L.W. Mints [in thesis acknowledgement Mints as the doctoral committee chair]
McKean, Roland [Neely] 1948 Lloyd W. Mints, chr.
Lloyd A. Metzler
Earl J. Hamilton
A. Director
Reeve, Joseph [Edwin] 1939 Lloyd W. Mints, chr.
Garfield V. Cox
Jacob Viner
Shaw, Ernest [Ray] 1930 Lionel D. Edie, chr.
Lloyd W. Mints
Stuart P. Meech (Bus. School)
Snider, Delbert [Arthur] 1951 L. Metzler, chr.
R. Blough
Bert Hoselitz
Tongue, William [Walter] 1947 L. W. Mints, chr.
Frank H. Knight
H. Gregg Lewis

            As you can see in some instances the Chairman was not listed, but the examining committee was listed. I wrote to Professor Cox, 660 W. Bonita, Apt. 24 E, Claremont, California 91711, to get the committee members for him and for Professor E. Shaw. Professor Cox also gave me the address of Professor Lloyd W. Mints, 618 E. Myrtle St., Ft. Collins, Colorado, should you have any interest. I hope this is sufficient.

Yours truly,
[signed]
(Mrs.) Hazel Bowdry
Sec. to Professor Telser

*  *  *  *  *  *  *  *  *  *  *

The University of Chicago
Chicago, Illinois 60637

Department of Economics

October 23, 1968

Professor Don Patinkin
Department of Economics
The Eliezer Kaplan School of
Economics and Social Sciences
The Hebrew University
Jerusalem, Israel

Dear Professor Patinkin:

            In answer to your letter of October 4, I have rechecked the files and find the below listed information.

George Bach’s committee members:

L. W. Mints, chr.
S. E. Leland
C. W. Wright
Oskar Lange
F. H. Knight
H. C. Simons
Jacob Viner
Jacob Left
Maynard Krueger

This is the order in which the examining committee is listed.

Martin Bronfenbrenner:

Henry Schultz chr.
J. Viner
L. W. Mints
F. Knight
A. G. Hart
H. C. Simon

Joel Harper:

S. E. Leland, Chr.
H. Simons
L. W. Mints
Mr. Chatters

Benjamin Brooks:

L. Mints, chr.
J. Viner
F. Knight

            I checked Faculty records with Mrs. Mosby, and found a re-appointment for Henry Simons dated June 3, 1930.

            I hope this information is helpful, and I am sorry I cannot give more definite committee members in the case of Bach.

Sincerely yours,
[signed]
(Mrs.) Hayzel Bowdry

P.S. I hope you have received the microfilm by now. It was mailed via airmail yesterday.

Source: Duke University. David M. Rubenstein Rare Book & Manuscript Library, Economists’ Papers Archive. Don Patinkin Papers, University of Chicago School of Economics Raw Materials, Box 2, Folder “Chicago, general (?), Simons, Mints, Knight materials”.

Image Source: Don Patinkin article at Gonçalo L. Fonseca’s History of Economic Thought website. Colorized at Economics in the Rear-view Mirror.

Categories
Chicago Economics Programs Economists

Chicago. Henry Simons’ Hayek project proposal, 1945

 

Henry C. Simons composed a dozen page, double-spaced, memo that he circulated in draft form to Hayek and the Chancellor of the University of Chicago, Robert M. Hutchins in May 1945. He was afraid that socialists and Keynesians (i.e. the Cowles Commission) were getting the upper-hand and that “traditional-liberal” economists like himself were becoming an endangered species. Not trusting university governing structures, Simons hoped to established an Institute of Political Economy that would dock onto the university but remain an independent beacon of traditional-liberal economics. 

I presumed the unnamed angel in all this was the William Volker Fund, but David Levy thinks the Earhart Foundation would have been a more likely addressee, given the list of people named by Simons. I find it curious that Simons never explicitly mentions a target foundation for his proposal though he had no reservations about including a long list of names of the economists he expected to support the work of his proposed Institute of Political Economy.

Hutchins wrote back to Simons in early September 1945, “I understand from the angel that Hayek has submitted another program, which has no relation to economics.” Simons’ proposal can be considered to have been an elevator pitch for a Chicago-based pre-Mont-Pèlerin Society.

Pro-tip.

According to the University of Chicago Archive’s Guide to the Henry C. Simons Papers, 1925-1972, Box 8, Folder 9 contains Simons’ file regarding his “Institute of Political Economy” proposal. The material for this post all come from Office of the President. Hutchins Administration Records. Box 73, Folder “Economics Dept., 1943-1945”.

______________________
Some of the Backstory

Henry C. Simons Urges his Department Chair (Simeon E. Leland) to Recruit Milton Friedman

August 20, 1945

Henry Simons’ grand strategy was to seamlessly replace the triad Lange-Knight-Mints with his own dream team of Friedman-Stigler-Hart. He feared that outsiders to the department might be tempted to appoint some convex combination of New Dealer Rexford Tugwell and trust-bustin’ George W. Stocking Sr., economists of the institutional persuasion who were swimming on the edges of the mainstream of the time.

______________________

Cover memo from Henry Simons to Robert M. Hutchins

THE UNIVERSITY OF CHICAGO

Date: May 19, 1945

[To:] Robert M. Hutchins

[From:] Henry Simons[,] Department [of] Economics

In re Hayek project

I enclose copies of two memoranda sent to Professor Hayek and of the covering letter.

Hayek asked Friedrich Lutz, Aaron Director, and me to send him suggestions and, when possible, to discuss the matter with one another. Other copies of the enclosures have been sent to Lutz, Director, and a few local people.

When you find time to look at this stuff, you might first read the letter and Memorandum II. The other item (Memorandum I) is long, discursive, and suitable, at best, only for very restricted circulation.

[signed]
Henry Simons

______________________

Henry Simons letter to Friedrich Hayek
[Carbon copy]

 

May 18, 1945

Professor Friedrich Hayek
London School of Economics
The Hostel, Peterhouse
Cambridge, England

Dear Professor Hayek:

I have been struggling to formulate a worthy and promising project that might attract endowment funds. Enclosed find two memoranda which are the poor results of my efforts. Memorandum II is mainly just a condensation of I—and is perhaps better suited for strangers.

I have departed very far from the kind of project we discussed here. I cannot muster or sustain much enthusiasm for any short-term project, or for any project which aims merely at another book or series of tracts. So much good money and professional effort has been wasted on such enterprises. My guess is that one should be less diffident about proposing what one really wants—that one might get both more (and “better”) money and fare better results by projecting something which the active participants might undertake and pursue with conviction and enthusiasm. Honesty is probably the best policy, even when seeking endowment funds.

I have contrived a project largely for what one might call ulterior purposes: (1) to get Aaron Director back here and into a kind of work for which he has, as you know, real enthusiasm and superlative talents; (2) to effect an arrangement regarding visiting professors which I have long espoused. Moreover, I have deliberately formulated the kind of project for which this University would be the natural location and for which Aaron would be a natural choice as head. But I doubt if such ulterior purposes condemn the scheme; on the contrary, the best procedure probably is that of making new schemes to do old things that one has long regarded as desirable. Indeed, the new device, as regards the stream of visitors, has very special merits, for it permits a continuity in the contribution of the visitors which could hardly be achieved otherwise.

I am sorry to have organized Lutz out of the picture—and hope he might be “organized in” again from time to time or permanently. He is probably the best choice for your kind of project; but Aaron seems a better choice for mine, if only by the nature of his own preferences and interests—although Lutz, in turn, would be a better choice for my project if it were located at Princeton.

My scheme may have little or no appeal to the particular donor. I’ve gotten too intrigued with formulating a project to give attention to its saleability to any individual.

We’re still sad about having seen so little of you and about having failed to keep you on for the Summer.

Cordially,

Henry C. Simons

HCS:w
Encl. 2 [Note: only memorandum 1 is to be found in the Hutchins file]

*  *  *  *  *  *  *  *  *  *  *  *  *  *

Memorandum I on a proposed
INSTITUTE OF POLITICAL ECONOMY

It may clarify all that I have to say here if I start with confession of my personal interests and selfish purposes.

A distinctive feature of “Chicago economics,” as represented recently by Knight and Viner, is its traditional-liberal political philosophy—its emphasis on the virtues of dispersion of economic power (free markets) and of political decentralization (real federalism for large nations and for supra-national organization). With the scattering of the “Austrians” and the vastly changed complexion of economics at Cambridge and Harvard, this intellectual tradition (of Smith, Ricardo, Mill, Menger, Wieser, Sidgwick, Marshall, Pigou, Clark, Taussig, Fisher, and Fetter, and of Locke, Hume, Bentham, de Tocqueville, von Humboldt, Acton and Dicey) is now almost unrepresented among the great universities, save for Chicago; and it may not long be well represented at Chicago. It has still many firm adherents, to be sure; but its competent representatives are widely dispersed and isolated from one another, in academic departments or governmental bureaus where they are largely denied opportunity for cooperation with like-minded scholars, or for recruiting and training their successors.

There should, I submit, be at least one university in United States where this political-intellectual tradition is substantially and confidently represented—and represented not merely by individual professors but also by a small group really functioning as a social-intellectual group. This objective presents difficulties, to be sure. Universities will seek to maintain balanced representation of major schools of thought (if not every fashionable novelty), in economics as in other departments; a group of traditional liberals large enough to function effectively might either dominate unduly any single economics department or require, for adequate representation of other “schools,” a department of excessive size. Moreover, traditional labels, individualists in political ideology, tend also to be lone -wolves and excessively individualist in their social-intellectual activities. More than other economists, they must, for real group activity, be selected with regard for their individual propensities for working with one another; if not inordinately friendly and congenial as persons, they are likely to go their separate ways, instead of cooperating, even if propinquity invites a more fruitful community activity.

Consequently, I see much merit in planning for such a group—for such a small social organization of traditional-liberal economists—without total reliance on departmental or university policy and with some loosely or informally affiliated “center” or “institute.” A few traditional-liberal professors might then function both as members of university departments, representing a suitable variety of schools or ideologies and not overlarge, and also as members of a different group centering around the small “institute” or “center” and organized deliberately in terms of a political philosophy or ideology.

Such an institute (Institute for Political Economy) should have a permanent head (Mr. Aaron Director). It should offer services, especially stenographic and mimeographing, for its local group. As its main function, it should, normally in cooperation with the university and department(s), arrange and partly finance extended visits of the best economists and political philosophers of its “school” from all over the world, one or two at a time. It might arrange local lectures or seminar talks by such economists when they happen to be passing through the city. It might sponsor a small local discussion club for faculty, advanced students, and selected outsiders. It might offer a few special fellowships for advanced study—for traditional-liberal economists (teachers, bureaucrats, journalists) as we now offer them for agricultural economists. It might help finance the writing and research of a few cooperating economists not visitors here. Above all, however, it should facilitate the group activity of the interested local professors and maintain a steady flow of competent visitors. From all its activities, a better flow of publications, both scholarly and semi-popular, might be anticipated; but this result should be planned by indirection—stimulated or facilitated rather than required under contracts with participants.

The permanent head of the Institute should be a broadly competent economist, with a major interest in a political philosophy and 19th century English political economic thought. He should be young enough to do creative work and yet mature enough to assure against his stepping out of character as a libertarian. He should be an essentially intellectual person, not a promoter, not politically ambitious or “on the make,” not “the administrator type,” not prominently identified with other organizations or public activity, and not adept at salesmanship or public relations. Indeed, the Institute should have no organized “public relations” at all, should cultivate obscurity, and, while promoting some popular writing, should seek primarily to make its influence felt in the best professional and academic circles, and merely by improving the quality of the writing (and teaching) of individuals. It should not ordinarily engage in publication or seek to identify itself in connection with the publications of its members or participants. Its head should be simply one scholar among scholars, seeking to hold together a group of individuals characterized by common political-economic persuasions, and to help them to help one another—by free interchange of ideas, mutual criticism of preliminary manuscripts, etc.

An important function of the Institute, indeed, should be that of providing typing, mimeographing, and mailing services for affiliated economists. It might facilitate organized discussion (1) of what people intend to write about, (2) of what they have prepared as tentative drafts, and (3) of what they are about ready to publish. Such discussion, besides stimulating writing, should greatly improve its quality, enabling an individual, before publishing, to thresh out disagreements with competent colleagues or, at least, to recognize what their disagreements or dissents are.

The most obvious merit of the scheme, for the University, lives in the plan of bringing in, for extended visits, the best available libertarian economists from other institutions and other countries. Such visitors might mainly or largely be younger men considered more or less eligible for regular appointment to the University faculty. In many cases, the University might be able to “look over” such men without the usual awkwardness of that process—to have them around for six or twelve months without any implied commitment to retain or even to “consider” them for permanent appointment. I should hope that the Institute would, in effect, deeply influence appointments to the faculty, merely by bringing excellent persons whom everyone, knowing them by their visit, would recognize as desirable appointees. It might also improve appointments by itself making this community more attractive to the best candidates.

The closest cooperation between the Institute and the University in the selection of visitors should be maintained. For distinguished visitors nominated by institute, the University might occasionally bear all, and often half, of the cost. For prospective appointees, the University might occasionally use the Institute as a dummy, thus getting a look at the candidate with a minimal [sic] of involvement and without risk of building up expectations that might be unpleasantly disappointed. Normally, it might be hoped that visitors would nominally divide their time between the Institute and the university, each bearing part of the cost.

I naturally would choose Chicago as a location for such an institute, and the University of Chicago as the institution with which to associate it. More substantial reasons than my personal predilections, however, could be offered for this choice. “Chicago economics” still has some distinctively traditional-liberal connotations and some prestige. Here, more than elsewhere, the project would be that of sustaining or keeping alive something not yet lost or submerged—and something which here, too, will shortly be lost unless special measures are taken.

However, I am somewhat open-minded about the location—and should myself be more than ready to go elsewhere, even at financial sacrifice, in order to participate in the kind of intellectual community in question. Likewise, I suspect that many able people might be attracted, at moderate stipends, to any good university where such a prospect was reasonably assured.

And I will concede that the outlook at Chicago, if better than elsewhere, is not very promising. Our Divisional dean has no appreciation of economic liberalism and a distinct hostility toward it, and the same is true of most persons in the other social science departments. Among higher administrative offices, there is at best only indifference, or provisional toleration, toward such political economy. A few members of the Law School and School of Business are interested or sympathetic, as are other individual faculty members here and there. In the department, moreover, we are becoming a small minority. Since I came to the University (1927), only one economist has been appointed who could be classified as really a traditional liberal (he, at an age when cure might still be anticipated); and one (the only fellow I ever found eminently useful as a colleague) was fired simply because of his uncompromising, competent profession of that political-economic philosophy. Meantime, many appointments have been made to the divisional economics staff; and a large staff, overwhelmingly hostile to economic liberalism, has been built up for the College courses in social science. Then, too, we acquired the Cowles Commission and its staff—whose influence the proposed Institute might partially neutralize or offset. Finally, there are our new agricultural economists who, while sympathetic, are real libertarians only avocationally.

Within the large department, there are now Knight, Mints, Viner, myself, and Lewis (in order of age). Knight will soon reach retirement age; Mints is not far behind; and Lewis, long frequently on leave, may well be attracted elsewhere. Moreover, Knight and Viner, while the best of libertarians, can hardly be called members of our group. Knight is increasingly preoccupied with the philosophy and philosophers, not to mention historians, theologians, anthropologists, et al., and is not deeply interested in concrete problems of economic policy. And Viner, while eminently useful to us as Journal editor, seems increasingly to dissociate himself both by interests outside economics and by very special preoccupations in his own writing and research. That leaves Mints and Simons to talk with and to stimulate one another, and to represent libertarian economics on the main teaching front—along with Lewis when he is here. (Viner and Knight teach only quite advanced courses and, even at that level, reach most of the students only in courses which stress technical matters, not political philosophy or political economy.)

On the other hand, our socialist and Keynesian colleagues are friendly and unusually tolerant toward us; and the others are not so much opposed to our political persuasions as simply uninterested—politically neutral or agnostic. It is a group which would be mainly friendly and cooperative with the Institute and its guests; it would doubtless welcome cordially most of the people whom the Institute would propose as visitors, and be happy to use the Institute occasionally for looking over possible appointees. No hostility would be likely to arise if the Institute was properly handled (for its own purposes) and if its resources were moderate.

Let me now formulate more concrete proposals.

(1) The Institute should be projected for roughly a 20-year period.

(2) It should have a permanent head (Aaron Director) with a salary of $7,500—the only person for whom the Institute would hold out permanent, full-time, professional employment.

(3) It should occupy a suite of three or four rooms at 1313 East 60th Street—or, like the Cowles Commission, on the campus—one for the director, one for a secretary-stenographer (or two?) and one for its visiting economists.

(4) It should plan to have one visiting economist (or political scientist, if libertarian ones can be found) on the ground all the time (save for its vacation periods)—and more than one if and as joint appointments and joint financing with the University are arranged.

(5) Finances permitting, it might grant a few fellowships (of, say, $1,000-$1,500) for the advanced training (or refresher training) of persons teaching economics at other institutions, or of interested practicing bureaucrats and journalists.

(6) It might also occasionally bring in outsiders for specific projects of writing and/or research—or assist them in completing publishing work done elsewhere.

(7) It would be highly desirable to have, in addition to the permanent head, a permanent half-time economic statistician, if arrangements could be made for joint appointment, with some department or school of the University, of a suitable person (e.g., Mr. Milton Friedman).

(8) In addition to one or two stenographer-secretaries, generous budgetary provision should be made for peak-load typing and for mimeographing of the manuscripts of economists affiliated with the Institute.

(9) These tentative proposals contemplate a budget of $20,000-$40,000 per year. A start could be made with less than $20,000, and more than $40,000 could easily be utilized effectively; but I distrust munificent arrangements. The important thing financially is assurance of continuity for a considerable period; but, again, I should urge against initial provision for more than 20-25 years. All this implies endowment of $300,000-$600,000—or assurance that funds of that (initial) present value will be steadily available.

The Institute should be set up, not as part of the University of Chicago but independently, with its own governing body and its own funds. It should be located at Chicago, however, only after reasonable assurance of close and friendly relations with the University; and it should be free to move elsewhere if effective or fruitful cooperation later proves unattainable here. The University might undertake to handle Institute funds; it should extend full use of facilities like the Library to the Institute’s director and its guests; it should offer facilities for lectures and seminars sponsored by the Institute; and it should undertake, when feasible, to make temporary (and perhaps one permanent ) joint appointments, so that guests of the Institute might also commonly serve also as members of the faculty. Close administrative cooperation and consultation should be continuously maintained. Cooperation, however, should be achieved largely through individuals, rather than by formal organizational connections.

The Institute should be designed primarily to promote cooperation and communication among competent economists of a traditional-liberal persuasion. It should aim to make such economists more cohesive and more articulate as a group. Its primary concern should be that of contributing to professional discussion and publication at the highest professional level, not that of popularizing or of propagandizing at a mass level. It may be hoped that such publication of popular or semi-popular books and articles would incidentally come about; and some direct efforts to this end would be appropriate. The Institute should seek to focus attention, not only on general economic-political philosophy, but largely on real, concrete problems and issues of public policy. It should, however, adhere firmly to a long and large view of policy, seeking not to influence immediate political action but to improve the quality of discussion of immediate matters. It should largely ignore considerations of immediate political expediency, seeking by discussion to influence professional opinion and thus perhaps to determine what will much later become politically feasible.

The director might properly occupy himself considerably with projects of non-technical writing on major policy problems. He might occasionally arrange for symposium publications, or for a series of special studies, with subsequent summary publications, for a wide audience. In the main, however, the director should be simply one member like others in an academic-intellectual community, contributing his share of talks and manuscripts to the common pool for mutual stimulation and criticism. Like others, moreover, he should publish mainly as an individual.

There are presumably plenty of agencies for publishing and disseminating good popular books and tracts. The Institute might quietly call attention to such writings of libertarian economists as might appeal to other organization; and it might occasionally subsidize or “undisclosedly enterprise” good publications which fail to find other outlets. In the main, however, it should seek to promote work which, when ready for publication, will readily attract commercial publishers. Its subsidies should be largely confined to unusual manuscripts which promise important contribution to professional discussion but do not promise commercially adequate sales.

The Institute, avoiding publicity, should be frank about its purposes and about its ideological position. Its director, its governing board, and all of its consulting or affiliated economists should be chosen as ardent, confirmed free traders—as anti-collectivists, anti-syndicalists, anti-“Planners”—as advocates of free foreign and free domestic trade, of non-discriminatory commercial policies, of untied, non-governmental foreign lending, of deorganization of functional groups, of deconcentration of economic power, of decentralization in national government, of impairment of national sovereignty (through supra-national organization), of devolution of central government powers (in favor of provisional and local powers); i.e., as advocates of systematic and progressive dispersion of power, nationally and internationally. They should be proponents of rigid economy in the kinds of governmental control or intervention—yet more concerned to minimize the kinds than the aggregate amount, and more concerned about minimizing the amount in large or central governments than in local and provincial bodies. Their central credo, following Acton and de Tocqueville, should be that no large organization can be trusted with, or wisely permitted, much power. They should be zealous proponents of the rule of law, of rules of policy as against legislative nose following, and of minimal delegations of discretionary authority. In a word, they should be confirmed constitutional-federalists in the strict sense.

That such an Institute would serve its proper or original purposes cannot be assured for a long period. It can be reasonably assured for (say) twenty years only by the most careful selection of personnel. One can trust Aaron Director to serve such purposes faithfully and intelligently. One can so trust Friedrick [sic] Hayek, Jacob Viner, Frank Knight, Lloyd Mints, Gregg Lewis, Theodore Yntema, Theodore Schultz, Garfield Cox, Wilber Katz, Quincy Wright, Ronald Crane and, to mention some persons elsewhere, Friedrick [sic] Lutz, Herbert Stein, Leland Bach, George Stigler, Allan Wallis, Howard Ellis, Frank Dunston Graham, Frank A. (and Frank W.) Fetter, Harry G. Brown, Joseph Davis, Karl Brandt, Leo Wolman, William A. Paton, Clare Griffin, I. L. Sharfman, Leverett S. Lyon, Milton Friedman, Arthur F. Burns, Gottfried Haberler, Eugene Rostow, Lionel Robbins, Fredrick Bonham, Henry Clay, R. G. Hawtrey, T. E. Gregory, Arnold Plant, A. J. Baster, Colin Clark, Roland Wilson, Harold A. Innis, Carl S. Shoup, James W. Angell, Thurman Arnold, Harry D. Gideonse, Reginald Arragon, Albert G. Hart, John M. Clark and, among prominent business men, William Clayton, and, among journalists, Walter Lippman, John Davenport, and Sir Walter Layton. Many others might be named, and some of those named above could be fully trusted only as members of an otherwise well-selected company.

Aaron Director is not only the ideal person to head the Institute; he is available and would be willing to undertake the task even at financial sacrifice (which he should not be expected to make). He probably would accept the modest stipend compatible with a properly modest and unobtrusive organization. No serious problem should arise in recruiting an able and reliable governing body or a fairly sizable company of conscientious, interested economist-participants or sponsors.

The Institute, to repeat, should not be designed primarily or explicitly as an agency for preparing tracts or reports. It should not be mainly concerned with formal economic theory; neither should it engage substantially in empirical research. It should focus on central, practical problems of American economic policy and governmental structure. It should afford a center to which economist liberals everywhere may look for intellectual leadership or support. It should seek to influence affairs mainly through influencing professional opinion and by preserving at least one place where some political economists of the future may be thoroughly and competently trained along traditional-liberal lines. Money for such causes is perhaps not hard to get and is very easy to spend wastefully or harmfully. In the project here suggested, I can see little danger of miscarriage and real promise of very good results.

______________________

Memo from Merrill Mead Parvis [?] to Hutchins and Colwell

THE UNIVERSITY OF CHICAGO

Date: June 14, 1945

R.M.H. [Robert M. Hutchins]
E.C.C.  [Ernest Cadman Colwell, President of the University of Chicago from 1945 to 1951]

In re Hayek à la Simons

There is an element of fear in Mr. Simons’ presentation of the true faith in economics. It sounds very familiar to me. It weakens any enthusiasm I may have had for the Hayek project. When it is seriously suggested that the staff for the institute should be drawn from men already so old that there is no risk of any ideas entering their heads, the cause must be in precarious condition indeed. Instead of the title that Mr. Simons suggests, I would suggest “asylum for laissez faire economists.”

In the second place, it seems to me that Mr. Simons takes all the vigor out of the proposal: It should not do serious research; it should not produce books that would influence public opinion; but it should aim at being a small, social, intellectual community, effecting contacts and influencing professional opinion. There is an element of dilettantism in this whole proposal, as I read it, that makes it sound like the laissez faire economists dinner club.

The statement of its relationship to the University seems to me to be a very simple one, not altogether desirable. The institute would be a pressure and propaganda group on the edge of the University entirely outside the University’s control, organized for the purpose of forcing or leading the University to appoint orthodox economists. None of this sounds very good for the University to me.

Yours truly,
[signed]

[Guess: Merrill Mead Parvis (1906-1983), colleague of Ernest Cadman Colwell, Chicago Ph.D. 1944, appointed associate professor of New Testament at Emory. Note that Colwell left Chicago in 1951 to become vice president and dean of faculties at Emory University.]

“Colwell was a New Testament scholar of some note. A graduate of Emory University, he received his PhD from the Divinity School at Chicago in 1930. He served on the faculty of the Divinity School from 1930 to 1951. One of his most remarkable decisions was to veto the appointment of George S. Stigler in 1946 to the faculty of the Department of Economics, on the grounds that Stigler was too empirical. See Ronald Coase, “George J. Stigler,” in Edward Shils, ed., Remembering the University of Chicago: Teachers, Scientists, and Scholars (Chicago, 1991), p. 470.

Source: Ftnt. 359 in John W. Boyer The University of Chicago: A History (2015), p. 571.

______________________

Carbon copy

Follow-up Memo from Hutchins to Simons

June 20, 1945

Dear Henry:

Thank you for the memoranda on the Hayek project. What has happened to this scheme?

Sincerely yours,

ROBERT M. HUTCHINS

Mr. Henry Simons
Department of Economics
Faculty Exchange

______________________

(Late) Reply to Hutchins by Simons

THE UNIVERSITY OF CHICAGO

Date: September 4, 1945

Chancellor R. M. Hutchins
From: Henry C. Simons [,] Economics [Department]

I am not remiss in telling you about the Hayek project, for there still is no further news. I have heard nothing from Hayek since he was here—which suggests either that he didn’t like my memos or that he has been preoccupied, possibly as a consultant on the treatment of Germany. Probably something unexpected has happened, for others have heard nothing from him; he is usually more than polite and “correct” about correspondence.

The memos and their scheme, however, were obviously not well contrived to get money from his particular “angel.” I had hopes that they just might be otherwise useful. Now that Sociology and Political Science are going into economics on their own, some scheme like mine is really needed as a counterpoise—not to mention E.H. Carr!

I’m taking the liberty of enclosing copy of a recent memo. [Not found in this file] Let’s hope it is not too irregular to do so, and that you will not be annoyed by passages which, at worst, were not intended to annoy you. Sending copy to you is an afterthought.

[signed, HCS]

HCS-w

P.S. A letter has just come from Hayek. Copy will go to you when it has been deciphered.

______________________

[Carbon copy]

Hutchins’ Reply to Simons

September 10, 1945

Dear Henry:

I understand from the angel that Hayek has submitted another program, which has no relation to economics.

What is the matter with E. H. Carr? I take few exceptions to your memorandum on Economics. My most important one is the implication that the Department is engaged in a bitter struggle with the administration to secure its just desserts. The administration would like nothing better than to make as many first-class appointments in Economics as the Department can prove are first-class.

The implication that the administration has put on pressure for “less good” appointments will prevent the administration from passing on without comment suggestions which it receives from reputable quarters. The suggestion of Stocking came from Edward H. Levi and was sent to Mr. Leland with no comments except those of Mr. Levi.

There is a kind of particularistic flavor about these suggestions for developments in connection with the Cowles Commission, the Law School, and possibly the School of Business, which imply that these are in the central field, whereas Industrial Relations, Agricultural Economics, Political Science and Planning, and possibly American Economic History are not. Some day I want you to explain to me why some of these areas are central and others are ancillary.

But what I started out to say was that I am glad that you are thinking about and pushing for the development of Economics in the University.

Sincerely yours,
ROBERT M. HUTCHINS

Mr. Henry C. Simons
Social Science 516
Faculty Exchange

______________________

The University of Chicago
Department of Economics

October 6, 1945

Chancellor Robert M. Hutchins
Faculty Exchange

Dear Mr. Hutchins:

Your good letter of September 10th was forwarded to me on vacation; hence the tardiness of this reply.

I share most of your disagreements with me! That memo was written for a small group of immediate colleagues—not hypocritically, I hope, but with “slants” that others might easily misinterpret.

I certainly have not felt that the Department is engaged in a bitter struggle with the administration to secure its just desserts. Neither do I object to the passing along of suggestions from reputable quarters. (Levi’s suggestion, by the way, was not without merit, if interpreted as part of his proposal for a large-scale local project in anti-trust investigation.) I was complaining about departmental policy or practice of making no longer-range proposals for recruitment and replacement—not about suggestions coming down to us but about the dearth of suggestions going up.

The Department, I think, should submit to the administration, not only recommendations for immediate, urgent appointments but also a “waiting list,” subject always to revision, of several men whom we definitely want if and when the administration is prepared to act on them. The administration might then make careful, unhurried outside inquiries; and, when outside suggestions are received, we might discuss and report on the relative merits of particular appointments and invite your inquiry on the same basis. Thus the waiting list or appointment program might be kept more or less continuously under critical discussion.

On that matter of what is central and what is ancillary, I think I have an important point, although I might have trouble stating it clearly or persuasively. The point, moreover, is one on which I would anticipate support from you.

About E. H. Carr, I am too strongly and deeply prejudiced for judicious comment. I have seldom reacted so strongly against a book as against his The Conditions of Peace—which is the only Carr book I have read. Knowing nothing of his work on Dostoevski or Bakunin, however, I would have less reason to oppose the appointment if it were in the proposed Russian Institute than if it were in Political Science and International Relations.

My objections to Carr are largely ideological. The Conditions of Peace is a powerful book, very well written and admirable in many parts and aspects. But it is largely and deeply concerned with economics and commercial policy; and here my criticism involves more than bitter disagreement; for here, I think, the fellow is using his rhetorical, journalistic skill to cover up his own lack of insight and understanding. One should not expect all students of politics to discuss economic problems competently. But one may object to their writing arrogantly, caustically, and demagogically about men, books, and subjects that they do not understand.

This book, I think, is one of the outstanding anti-Liberal documents of its time, not only as regards economic policy, domestic and international but also as regards the rights of small nations and their proper place or role in the good society. Carr personifies, for me, almost everything that is wrong with political thinking at both the extreme Right and the extreme Left.

It is significant, I think, that Carr has earned the most bitter denunciation of two such different people as Hayek and Keynes. (Don’t quote me as regards Keynes, for my information is somewhat privileged in that case and second-hand; but I believe it may easily be confirmed.) At best, Carr is a very hot potato in present-day politics—much too hot for wise University appointment, even if one approved of his views.

I should be more diffident about my own reactions to Carr if those of J. Viner and Q. Wright (and Louiee Wright) were not much the same. Incidentally, what is distinctive about Carr (tough political “realism”) is, I think, already adequately represented here, and competently, by Morgenthau.

I’ll be happy to talk sometime about what is central what is ancillary—or as happy as I can be when trying to talk philosophically,

Sincerely yours,

[signed] Henry Simons
Henry C. Simons

ECS-w

P.S. I hear that Milton Friedman, whom I was proposing for Lange’s place, has been appointed to an associate professorship at Minnesota. My scheme thus requires raiding the Minnesota staff for two men, within a few years. Moreover, it might now be best, under that scheme, to get Stigler first.

Source:  University of Chicago Archives. Office of the President. Hutchins Administration Records. Box 73, Folder “Economics Dept., 1943-1945”.

Image Source:  Henry Calvert Simons portrait at the University of Chicago Photographic Archive, apf1-07613, Special Collections Research Center, University of Chicago Library.

 

Categories
Chicago Economists Salaries

Chicago. Selected salaries. Hayek visiting, Friedman as associate professor, 1946

 

 

Since economists put much store in the notion of people putting their (own or other people’s) money where their mouths are, Economics in the Rear-view Mirror provides from time to time some historical faculty salaries to shine a little light on where those professors of economics before us stood in the willingness-to-pay of their respective departments and university administrations. In this post we see how the brief visiting professorship of Friedrich Hayek and the tenured associate professorship of Milton Friedman fit into the 1946 salary structure at the Univerity of Chicago’s department of economics.

Note: For his half-quarter service Hayek was offered $2,000 (quoted in a January 23, 1945 note  from the director of the U of Chicago Press to VP E. C. Colwell). I presume the $4,000 figure includes $2,000 compensation from (or on behalf of) Stanford University.

_______________________

Comparison: Selected 1945-46 Chicago Salaries
(and recommendations for 1946-47)

Jacob Viner. $10,000
Frank Knight. $9,000 ($10,000)
S.E. Leland. $9,000 ($9,500 Note: resigned to go to Northwestern)
T.W. Schultz. $9,000 ($9,000)
John U. Nef. $8,000 ($8,000)
Jacob Marschak. $8,000 ($8,500)
Paul H. Douglas. $7,000 ($8,000)
Oscar Lange. ($6,000) ($6,000) on leave 1 Oct 1945 to 30 June 1947
Henry Simons. $6,000 ($6,000)
L. W. Mints. $5,500 ($6,000)
Tjalling Koopmans $5250 ($6,740. Note: new salary effective 1 January 1946)

Source:  “Budget and Appointment Recommendations 1946-47 (December 7, 1945)”

_______________________

Hayek’s Half-Quarter, Spring 1946

 

May 10, 1946

Mr. Robert Redfield Social Sciences
R. G. Gustavson Central Administration

On May 9, 1946 the Board of Trustees approved the following recommendations:

It is recommended that Friedrich A. Hayek be appointed Visiting Professor of Economics in the Department of Economics for the period April 8, 1946 to May 11, 1946. For this service and a similar period of service at Stanford University it is recommended that an honorarium of $4,000 be approved.

cc:
Mr. T. W. Schultz
Mr. L. A. Kimpton)      Salary not mentioned
Mrs. K. Turabian)        Salary not mentioned

 

Board—5/9/46:

It is recommended that Friedrich a. Hayek be appointed Visiting Professor of Economics in the Department of Economics for the period April 8, 1946 to May 11, 1946. For this service and a similar period of service at Stanford University it is recommended that an honorarium of $4,000 be approved.

Form sent to Comptroller—5/13/46

*  *  *  *  *  *  *  *  *

Milton Friedman’s tenured associate professorship
Effective October, 1946

March 19, 1946

Mr. Robert Redfield Social Sciences
R. G. Gustavson Vice President

On March 28, 1946 the Committee on Instruction and Research approved the following recommendation:

It is recommended that Milton Friedman be appointed Associate Professor of Economics in the Department of Economics on indefinite tenure on a 4E Service basis at an annual salary of $6,000 effective October 1, 1946.

cc:
Mr. T. W. Schultz
Mr. L. A. Kimpton)      Salary not mentioned
Mrs. K. Turabian)        Salary not mentioned

 

I & R. 28 March 1946:

It is recommended that Milton Friedman be appointed Associate Professor in the Department of Economics on indefinite tenure on a 4E service basis at an annual salary of $6,000 effective October 1, 1946.

 

Source: University of Chicago Library. Department of Special Collections. Office of the President. Hutchins Administration Records. Box 284. Folder “Economics, 1943-1947”.

Image Source: National Portrait Gallery. Photographs Collection. NPG x187289. Friedrich August von Hayek by Walter Stoneman, half-plate glass negative, June 1945. The portrait has been cropped to fit the format of this webpage.
Creative Commons License Creative Commons license. Attribution-NonCommercial-NoDerivs 3.0 Unported (CC BY-NC-ND 3.0).

Categories
Chicago Exam Questions Fields Undergraduate

Chicago. Comprehensive Exams in Economics for B.A., 1941

 

 

One presumes that a departmental comprehensive examination would cover material that would be expected of any student going on to graduate studies in economics.  The comprehensive examination for Harvard economics majors from 1953 has been previously posted as has Swarthmore’s comprehensive examination for 1931.

A few things worth noting:

  • Henry Simons and Paul Douglas were apparently enough at odds with each other’s economics to be unable to come up with a single principles examination in Part I.
  • Both accounting and basic statistics shared equally in the quantitative Part II.
  • Either U.S. or European Economic History was required to be one of the three field examinations in Part III. A student could even take both economic history examinations, so one can say economic history was very much part of the common core for economists-in-training.
  • From today’s perspective it is interesting to find that “transportation” was a field still having equal status with “labor” and “government finance”.

According to a handwritten note attached to the following comprehensive exam was used four times:  Spring 1940, Winter 1941, Autumn 1941, and (with slight correction) Winter 1942.

__________________

PART I

COMPREHENSIVE EXAMINATION FOR THE BACHELOR’S DEGREE IN ECONOMICS

(Start each new subject in a new examination book)

The comprehensive examination in Economics is divided into three parts:

PART I — Time: Approximately 2 ½ hours.

(a) Principles of Economics
(b) Principles of Money and Banking

PART II — Time: Approximately 2 ½ hours.

(a) Elementary Accounting
(b) Statistics

PART III — Time: Approximately 3 hours.

Write on either (a) or (b) and two other subjects. One of these may be the second subject in Economic History.

(a) Economic History of the United States
(b) Economic History of Europe
(c) Labor
(d) Government Finance
(e) Transportation

 

 

PART I

(a) Economic Principles

Write on either examination A or examination B. In view of the difference in reading lists, examination A is offered primarily for those who did their work in Economics 209 with Mr. Douglas, while examination B is for those who had this course with Mr. Simons.

Examination A.
(Answer all questions.)

  1. Describe in some detail why the demand curves for the products of an industry are negatively inclined and give and illustrate the formula for the measurement of elasticity.
    Why, under atomistic competition, is the demand curve for the products of an individual firm of infinite elasticity and indicate by graphs what forces determine equilibrium for the individual firms (a) with no alternation in their number, (b) in the longer run, where the numbers of firms may vary but where there is no change of the scale of the individual plant, (c) in the still longer run when both the numbers and the scale of plants vary.
  2. Discuss and illustrate equilibrium under conditions of “imperfect competition,” showing (a) the role of average and marginal revenue curves, (b) average and marginal cost curves. Discuss both short-run and long-run equilibrium and the light such conclusions throw upon whether competition is or is not desirable, the proper role of the state, etc.
  3. Trace the theory of production, showing the relative effect upon product of changes in the quantities of the three factors of production, i.e., land, labor and capital, and the steps by which the theory of distribution can be derived from the theory of production.

 

Examination B.
(Answer both questions.)

  1. (50 points)
    In an isolated community there are two kinds of land, and only one product, wheat. There are 100 farms of each The labor supply is homogeneous—i.e., all workers are equally efficient. There is private property in land and free contract for labor. Labor services are bought and sold only in units of one laborer per year. The markets for both labor and land (unless otherwise specified) should be assumed to be freely competitive.
    The table below shows the amounts of wheat which can be obtained from onesingle farm of each grade, with different numbers of laborers per year.
Number of Laborers Output on A-grade Farm Output on B-grade Farm
1 1,000 900
2 1,800 1,200
3 2,400 1,400
4 2,900 1,550
5 3,300 1,650

The labor population is 450 — all workers will seek to be fully employed at any wage rate above zero.

a. What will be the wages per man? Explain why.

b. What will be the rent of farms of each grade?

c. Explain how the productivity (product increment) of an A-grade farm may be determined.

d. What would happen to wages and rents if an output tax of 5 per cent were imposed upon the production of wheat?

e. What would happen to wages and rents if a tax of 100 bushels per farm were levied, the tax being payable by owners?

f. Suppose a minimum wage law is passed and enforced, requiring the payment of at least 700 bushels per year for labor. What will be the effect on total employment and on rents?

g. Suppose that workers on the A-grade farms organize into a trade union and enforce a minimum wage of 700 bushels per year on the A-grade farms. What will happen to rents? To numbers of workers employed on A-grade farms? To the wages of workers not employed on A-grade farms?

h. Suppose that workers organize only on the B-grade farms and enforce there a wage of 700 bushels per year. What will happen to rents? To wages on the A-grade farms?

  1. (50 points)
    Indicate the conditions or circumstances under which each of the following relationships is likely to obtain, in the short run if not in the long run, and explain briefly in each case:

    1. Marginal revenue is equal to price.
    2. Price is equal to average expense (total cost per unit) but far in excess of marginal expense.
    3. Marginal expense, for the industry as a whole, fare exceeds marginal expense for the individual firm.
    4. All firms in a highly competitive industry are maintaining outputs at which their average-cost curves are falling (negatively sloped).
    5. All firms in a highly competitive industry are maintaining outputs at which their marginal-expense curves are falling.
    6. The price of a productive service is equal to its product increment times product price.
    7. The price of a productive service is much less than its product increment times product price.
    8. The price of a productive service is much less than its product increment times marginal revenue (for the firm).
    9. The total output of all firms in an industry is such that marginal revenue, for the industry as a whole, is negative.
    10. Marginal expense and average expense are equal but both are far in excess of product price.

 

(b) Principles of Money and Banking

(Answer all parts in questions 1 and 2; if time permits answer question 3.)

  1. (25 points)
    The following statements are to be completed by filling in the blanks with the most nearly correct of the suggested answers:

    1. Excess reserves of the member banks of the Federal Reserve System are currently about _______ million dollars. (100; 1,000; 1,500; 3,500; 18,700)
    2. The Federal Open Market Committee consists of _______ (5; 7; 9; 12;19) members, of which (1; 3; 5; 7; 12) are members of the Board of Governors of the Federal Reserve System and the remainder selected by ____________________ (President of the U.S.; Board of Governors; U.S. Secretary of the Treasury; directors of the Federal Reserve banks).
    3. In recent months holdings of U.S. Government securities (direct and guaranteed) by the Federal Reserve banks have totaled about _______ million dollars (25; 500; 2,500; 6,000).
    4. A member bank in downtown Chicago is at present required to hold with its Federal Reserve Bank an actual net balance equal to _______ (10; 13; 17½; 22¾; 26) per cent of its net demand deposits.
    5. If the U.S. Treasury were to shift its present deposits from member banks to the Federal Reserve banks, excess reserves of member banks would probably _______ (increase; decrease; remain unchanged) and excess reserves of the Federal Reserve banks _______ (increase; decrease; remain unchanged).
    6. The Board of Governors of the Federal Reserve System is authorized to decrease existing reserve requirements for reserve city member banks to a minimum level of _______ (5; 13; 17½; 20; 100) per cent against its net demand deposits.
    7. The total volume of hand-to-hand money in circulation in the U.S. (in the hands of the public and in banks’ vault cash) has recently been approximately _______ (600; 8,000; 10,000; 50,000) million dollars, of which approximately _______ (0; 5; 25; 30) per cent has consisted of gold coin.
    8. In recent years member banks have held approximately _______ (10; 25; 55; 85; 98) per cent of all demand deposits (excluding inter-bank deposits) in all commercial banks of the country.
    9. If the Federal Reserve banks sold their present holdings of U.S. Government securities to the public, excess reserves of banks in the country would probably _______ (increase; decrease; remain unchanged).
    10. In computing its demand deposits subject to legal reserve requirements, a member bank may deduct from its gross demand deposits _______ (U.S. deposits held with it; balances due from other domestic banks except Federal Reserve banks; its vault cash; balances due to other domestic banks).
    11. In giving a correct statement of the quantity theory of money, it is necessary to state among other things the assumption _______ (that wage rates remain constant; that the country is not on a paper monetary standard; that the economy to which it refers is perfectly competitive; that the theory may not be applicable in the short run).
    12. The monetary gold stock of the United States is currently approximately _______ (3.5; 7.0; 22; 25) billion dollars.
    13. Treasury purchases of imported gold will result in the greatest reduction in excessreserves of banks (not including Federal Reserve banks) when the Treasury pays for the gold by _______ (issuing new gold certificates; borrowing funds from the public; borrowing funds from commercial banks; borrowing funds from the Federal Reserve banks).
    14. Time and demand deposits (excluding interbank deposits) in all banks of the United States currently total about _______ (25; 40; 60; 75) billion dollars, of which amount approximately _______ (10; 25; 40; 60; 98) per cent is fully insured by the Federal Deposit Insurance Corporation.
    15. Under present conditions the Federal Reserve banks can most effectively reduce excess reserves of member banks by _______ (raising the discount rates of the Federal Reserve banks; selling their holdings of U.S. Government securities on the open market; raising the legal reserve ratios of member banks to 100%).
  2. (75 points)
    A recent annual report of the Board of Governors of the Federal Reserve System contained the following statement:
    “Under existing conditions the Treasury’s powers to influence member bank reserves outweigh those possessed by the Federal Reserve System.”

    1. State briefly and concisely the powers of the U.S. Treasury to influence member bank reserves; evaluate and explain their importance with reference to:

(1) Increasing member bank excess
(2) Decreasing member bank excess reserves.

    1. If the Treasury were to use certain of its powers, it could increase its cash holdings (without borrowing or taxing) by 10 billion dollars. Assume that it does so today, and that it spends the 10 billion dollars for national defense goods (in addition to the expenditures previously budgeted) during the next two years. Analyze the effects of the spending, including in your analysis statements concerning the effects on:

(1) Employment and national income.
(2) The cash position of the public.
(3) The reserve position of commercial banks.
(4) The powers of the Federal Reserve System to reduce member bank excess reserves.
(5) Relative changes in important groups of prices.

Of what help is the quantity theory of money to you in explaining the price fluctuations of (5)?

  1. (30 points)
    (If time permits)
    Defend your answers to parts e, I, m, and o of question 1.

 

 

PART II

(a) Elementary Accounting

(Answer all questions; plan to spend at least 40 minutes on question 4.)

  1. Debits and Credits
    Directions: Read the data given and select from the “Numbers To Be Used” the appropriate debit and credit to be used. Write the numbers of these accounts in the appropriate column, indicating in each case the kind of account (A-L-P-E-I).

Numbers to be Used

(1) Accounts Payable (10) Notes Payable
(2) Accounts Receivable (11) Notes Receivable
(3) Bad Debts (12) Office Expense
(4) Cash (13) R. Smith, Capital
(5) Furniture and Fixtures (14) Purchases
(6) General Expense (15) Sales
(7) Interest Cost (16) Wages and Salaries
(8) Interest Income (17) Rent Expense
(9) Merchandise Inventory

 

Debit Credit
Sample: A customer pays us cash on account (4) (A) (2) (A)
1. R. Smith invested cash in a mercantile business 1.
2. Paid cash for rent of store building 2.
3. Bought fixtures for cash 3.
4. Bought merchandise on account 4.
5. Bought office supplies for cash 5.
6. Sold merchandise for cash, note, balance on account 6.
7. Gave a trade creditor a note on account 7.
8. Paid a trade creditor cash on account 8.
9. Paid note payable due a creditor, with interest 9.
10. Received cash on account from a customer 10.
11. Received payment of note due from customer, with interest 11.
12. Paid wages and salaries 12.
13. Paid miscellaneous expenses 13.
14. A customer goes bankrupt and pays only a part of his account, the rest being uncollectible 14.
15. Bought merchandise for cash, note, balance on account 15.
16. Traded merchandise for furniture and fixtures 16.

 

  1. The following statements are to be marked by circling “T” if true, or “F” if false. A statement which is in any part incorrect is to be considered false.

T or F. The declaration of cash dividends results in a current liability on the balance sheet.

T or F. For a corporation having only common stock outstanding, the book value of the common stock is equal to the result obtained by dividing the difference between the total assets and the total liabilities by the number of common shares outstanding.

T or F. Customers’ accounts with credit balances should be shown on the balance sheet as current liabilities.

T or F. If the ending raw materials inventory is valued at too low a figure (other data on the statements correct), the cost of goods sold will be too small.

T or F. If depreciation of an asset is overestimated, that asset will be overvalued on the balance sheet.

T or F. A partnership is always automatically dissolved by the death of any one of its members.

T or F. Stock-dividends declared but not yet issued are shown on the balance sheet as current liabilities.

T or F. If all the stockholders of a corporation die, the corporation ceases to exist.

T or F. Holders of cumulative preferred stock have an unconditional right to dividends that are in arrears.

T or F. If the goods in process inventory at the beginning of an accounting period is overstated (other data on the statements correct), the gross profit for that period will be too small.

T or F. A corporation with a $200,000 surplus account could have no difficulty in paying a $100,000 cash dividend to stockholders.

T or F. Patents are written off to factory expense over the period of their economic life which cannot be more than 17 years.

T or F. Capital surplus represents the amount of profits which the stockholders and directors have been willing to leave invested in the business.

T or F. Expenditures which increase the usefulness of an asset, or prolong its life, are capital expenditures.

T or F. The introduction of controlling accounts for expenses makes necessary some change in the form of the journals used by that business.

T or F. Discount on Stock may be correctly shown on the balance sheet as a deferred charge.

T or F. A sinking fund reserve is set up to prevent the use of sinking fund cash for dividend purposes.

T or F. Preferred stock is never entitled to preference in the distribution of assets in liquidation, unless specified in the stock agreement.

T or F. A firm which has incurred a loss for the year may have more cash on hand at the end of the year than it had at the beginning of that year.

T or F. The cost of repairing a second-hand machine, before it is put to use in the factory, should be charged to factory expense.

 

  1. You are given a Statement of Profit and Loss of the Northwestern Manufacturing Company for the year ended December 31, 1940. Profit is shown as $121,380 Upon investigation you find that the accountant had proceeded as follows:
    1. Inventory had been valued at Market, $180,000; Cost was $150,000.
    2. Depreciation had been calculated on new machinery (purchased January 1, 1940) at a 10% rate. The general experience of competitors indicated that the life of the equipment was five years. The cost of the machine under question was $38,000.
    3. Wages due salesman for services rendered, $8000, had been overlooked.
    4. A garage owned by the Company was destroyed by fire. The building had a book value of $30,000. The insurance company had agreed to pay $20,000. The Company had signed a release but no record had been made of the fire or agreement.
    5. Accounts Receivable were valued at Gross, $200,000.
    6. Competitors had found that about 2% of gross accounts were uncollectible. About $1000 in cash discounts applicable to 1940 were expected to be taken.

What changes would you make on the Balance Sheet and the Statement of Profit and Loss for each of the above items?

  1. List the problems associated with the valuation of fixed assets: (a) at the time of acquisition, (b) of changes subsequent to the time of acquisition. Explain the relationship between these problems and cost determination in a manufacturing enterprise. Suggest solutions which the accountant has used in the past and discuss these critically in terms of economic theory.

 

(b) Statistics

(If time permits, answer all questions; note the unequal weighting, however. Plan to spend approximately 30 minutes on question 3.)

  1. (25 points)
    In the space to the left of each of the following statements indicate whether the statement is true (T) or false (F). Do not guess; if you don’t know whether a statement is true or false, don’t market.

_____ a. In a series of positive numbers the algebraic sum of the deviations of the individual items from their arithmetic mean is positive.

_____ b. In a simple linear correlation the slopes of the two elementary regression lines are always the same.
_____ c. Fisher’s Ideal Index Number formula satisfies both the time reversal and factor reversal tests.
_____ d. A moving average of points which lie along a straight line will reproduce the line.
_____ e. The sum of the squared deviations from the median of the frequency distribution is less than the sum of the squared deviations from any other average of the same frequency distribution.
_____ f. In simple linear correlation the two elementary lines of regression are identical if the simple correlation coefficient (r) is plus one and perpendicular to each other if the simple correlation coefficient is -1.
_____ g. The time series of the population of the United States plots is a straight line on semi-log paper; therefore, we may conclude that the population of the United States has grown at a constant relative rate.
_____ h. The simple correlation coefficient (ryx) is the arithmetic mean of the two simple regression coefficients (bxy and byx).
_____ i. In every frequency distribution 68% of the cases lie within plus and minus one standard deviation from the arithmetic mean.
_____ j. If the simple linear correlation coefficient between X and Y is small, it shows that there is very little relationship of any kind between X and Y.
_____ k. The standard error of estimate for the regression of Y on X depends upon the units in which Y is measured.
_____ l. The aggregative price index with base year quantity weights is identical to the arithmetic index of price relatives weighted by values of the base year.
_____ m. The sampling distribution of means of samples (all of the same size) drawn at random from a normal universe is also normal.
_____ n. The product of the individual items of a series of numbers is unchanged if each of the items is replaced by the geometric mean.
_____ o. The ratios-to-trend method of obtaining an index of seasonal variation is valid only if the underlying trend his linear.
_____ p. If the probability of getting a tail in a single toss of a bias coin is 1/4, the probability of getting three heads in three independent tosses of the same coin is 3/4.
_____ q. The sampling distribution of means of samples (all of the same size) drawn at random from a non-normal universe is less normal than the universe itself.
_____ r. The standard deviation of the sampling distribution of means drawn at random depends upon the size of the samples.
_____ s. The simple geometric average of relative prices satisfies the time reversal test.
_____ t. If a frequency distribution is symmetric when plotted on the arithmetic scale, the geometric mean, the median, and the mode will all coincide.
_____ u. If a frequency distribution is symmetric when plotted with a logarithmic scale on the X-axis, it will be skewed when plotted on the arithmetic scale.
_____ v. The harmonic mean of a series of positive numbers is sometimes greater in the geometric mean.
_____ w. The median is less affected than the arithmetic mean by the magnitude of extreme observations.
_____ x. The probability that two independent observations drawn at random from the same normal universe will both deviate by more than one standard deviation from the arithmetic mean of the universe is approximately 0.32 (= 32%).

  1. (35 points)
    State the reasoning behind your answer to the following parts (seven in all) of question 1:

(a or n)
(b, f, or h)
(c or s)
(i)
(j)
(p or x)
(r)

In each case, if you marked the statement true demonstrate its truth; if you marked it false, revise it so that it is correct, and demonstrate that your revision is true. Use mathematics where convenient.

  1. (40 points)
    The ABC Corporation which manufactures and sells over 1,000,000 packages of cigarettes (20 cigarettes per package) per year advertises of that on the average their cigarettes will burn for 15 minutes (per cigarette).
    The XYZ Corporation, making and selling over 2,000,000 packages of cigarettes per year (20 cigarettes per package) asserts that on the average its cigarettes will burn for 16 minutes (per cigarette).
    The Honesty-in-Advertising Association samples each manufacturer’s cigarettes, taking one sample of 145 cigarettes (not packages) of each Corporation’s. The following is a tabulation of their findings:
Maker of Cigarette Mean Burning Time
(in Minutes)
Sample of [sic] Standard Deviation of Burning Time
(in Minutes)
ABC Corporation 14.5 6.0
XYZ Corporation 15.0 4.0

On the basis of the above findings,

a. Do you feel that the claims of each manufacturer are justified?

b. Do you feel that XYZ cigarettes on the average burn longer than ABC cigarettes.In answering these questions make use of whatever relevant logical techniques you have learned. State your reasoning carefully; your reasoning is even more important than your arithmetic.
Note: The square root of 52 is 7.2.

 

 

PART III

Write on either (a) or (b) and two other subjects.
One of these may be the second subject in Economic History. (Approximately 3 hours).

(a) Economic History of the United States

(Answer the first three questions and, if time remains, the fourth.
Answer in outline form so far as possible.)

  1. Briefly describe or explain.

a. colonial indentured servant;
b. growth of slavery in the colonies;
c. coinage act of 1792;
d. rise of steamboats in the Mississippi Valley;
e. tariff of 1833;
f. railroad land grants of 1862-71;
g. transportation act of 1920;
h. War Industries Board;
i. Congress of Industrial Organization;
j. wages and hours act of 1938.

  1. Enumerate the chief causes for:

a. adoption of the public land act of 1820;
b. decline of canals after 1860;
c. decline of the general price level, 1865-1896;
d. shifted to a favorable balance of commodity trade after 1873;
e. restriction of immigration after 1921;
f. distressed condition of agriculture since 1920;
g. demand for a New Deal in 1933.

  1. Compare the chief exports and imports of about 1860 with those of the post-World War period. Carefully explain the chief economic developments responsible for the changes that took place.
  2. Outline and explain the history of the merchant marine, 1789-1940.

 

(b) Economic History of Europe

(Answer two questions.)

  1. Discuss the significance of any two of the following authors for the student of modern European economic history: Buckle, Tawny, Spengler, Clapham.
  2. Compare the role of the state in industrial enterprise in France and England during the seventeenth century. Did the French or the English government do the most for the general welfare of its people by its industrial policies?
  3. Compare the influence of either the railway or the canal upon the economic development of France, England, and Germany.

(c) Labor

(Answer both questions.)

  1. Discuss:

a. the main features of the various state minimum wage laws and the federal Fair Labor Standards Act;
b. the economic theories upon which they are based;
c. the constitutional issues involved.

  1. Discuss the issues involved as regards structure, membership, aims and methods in the following struggles:

a. The A. F. of L. versus the Knights of Labor.
b. The I.W.W. versus the A. F. of L.
c. Shop committees (or so-called employee representation plans or as sometimes termed “company” and “independent” unions) versus so-called “outside” unions.
d. The C.I.O versus the A.F. of L.

 

(d) Government Finance

(Answer all questions.)

  1. (35 points)

Mark each of the following propositions “True” or “False” and explain briefly (on separate paper):
The exemption, under federal personal-income tax, of interest on the obligations of state and local governments
_____ a. Involves a kind of federal subsidy or grant which is not commendable in terms of the basis on which the different states share relatively.
_____ b. Probably involve serious inequity as among large income receivers of similar income circumstances.
_____ c. Lowers the rate of interest which state and local governments must pay on their new borrowings.
_____ d. Probably serves to retard or delay recovery from severe depressions.
_____ e. Imposes indirectly a significant burden upon persons of small income in their capacity as savers.

  1. (25 points)
    It is often argued that income taxes, while having great merit in other respects, are ill-suited for a predominant place in revenue systems because their revenue-yield fluctuates so widely between years of prosperity and depression. Are such wide fluctuations a fault or virtue in a federal tax? Discuss.
  2. (25 points)
    In spite of its excellent cumulative features, the federal gifts tax leaves large opportunities for avoidance of estates tax through the distribution of property by gift. Explain “cumulative features”; and indicate the relevant facts about the law which have to do with the avoidance opportunities.

 

(e) Transportation

(Answer all questions. Note weighting of questions.)

  1. (10 points)
    In the following statements, underline the figure, or concept, that most nearly accords with accuracy.

    1. Operating expenses of a railroad may be expected to vary in accordance with:
      tons of freight carried; passenger-miles; train-Miles; car-mile; miles of track
    2. The standard gauge of American railroads is:
      3 ft. 6 in.; 4 ft.; 4 ft. 8 in.; 5 ft. 2 in.; 5 ft. 5 in.
    3. The average freight traffic density of American railroads is:
      100,000; 500,000; 1,000,000; 1,500,000; 5,000,000; 10,000,000
    4. The Interstate Commerce Commission was given power to prescribe actual railroad rates in:
      1906; 1903; 1887; 1911; 1920
    5. The carrying capacity of ocean ships is customarily expressed by:
      gross registered tons; deadweight tons; net registered tons; displacement tons; cargo tons of 40 cu. ft.
    6. The regulation of the rates of waterway common carriers in interstate commerce was authorized by Congress in:
      1900; 1916; 1920; 1933
  2. (15 points)
    The following diagram represents two railroad roots and 6 stations, the figures indicating the mileage between each pair of stations. The East and West Railroad serves all these points.

Indicate which of the rate situations stated below are departures from the provisions of the 4th Section of the Interstate Commerce Act:

a. A rate of 50¢ on commodity “X” from A to E, and 75¢ from E to B.

b. A rate of 25¢ on commodity “X” from A to B, and 20¢ from A to C.

c. A rate of 40¢ on commodity “X” from A to D, and 60¢ on commodity “Y” from A to C.

d. A rate of 45¢ on commodity “X” from A to C, and 50¢ on the same commodity from C to E.

e. A rate of 75¢ on commodity “X” from A to F via C, and 50¢ from A to F via E on the same commodity.

  1. (10 points)
    Draw up definitions of “common carrier” and “contract carrier” for the purpose of establishing a system of regulation of water carriers in interstate commerce of the United States.
  2. (20 points)
    The following diagram represents the line of a single railroad with 8 stations. The numbers represent the distances between stations:

Suppose that the rate structure on traffic between these points is represented by the 1st and 5th class rates, and commodity rates on furniture, and steel products, such as sheets, bars, rods.

From A
to
All rates are cents per 100 lbs.
1st Class 5th Class Furniture Iron and Steel
B 25 20 10 16
C 31 22 12 20
D 20 19 10 17
E 37 25 13 22
F 48 30 17 29
G 50 33 20 31
H 50 36 20 31

Assume neither water nor highway competition. What departures from principles of rate-making do you detect in this rate structure?

  1. (15 point)
    The Omnibus Transportation Bill which passed in the House of Representatives last Summer, inter alia, contain the following provisions: “In order that the public at large may enjoy the benefit and economy afforded by each type of transportation, the Commission shall permit each type of carrier or carriers to reduce rates so long as such rates maintain a compensatory return to the carrier or carriers after taking into consideration overhead and all other elements entering into the cost to the carrier or carriers for the service rendered…”Should such a provision be finally adopted into the law and seriously enforced by the Commission, what effect presumably would it have on the freight rate structures, and on the distribution of commodities? Why?
  1. (10 point)
    In which of the following cases is a certificate of public convenience and necessity required? Check the affirmative cases.

    1. A railroad desires to refund a maturing issue of bonds.
    2. Two motor highway common carriers wish to consolidate properties and operations.
    3. John Smith wishes to inaugurate a highway service between Chicago and St. Louis. He has a contract with a St. Louis manufacturer to haul enamel ware to Chicago; and this will take all his facilities northbound. But he desires to secure return loads and will haul any traffic that is offered.
    4. A railroad is about to acquire a new Diesel stream-lined train.
    5. A water common carrier, finding operations entirely unprofitable, decides to abandon operations.
  2. (10 points)
    A common carrier subject to the jurisdiction of the Interstate Commerce Commission files a tariff containing new schedules of rates, embodying a number of changes. Which of the following statements most accurately describes the Commission procedure in dealing with the tariff.

    1. The tariff is passed around among the 11 commissioners, each of whom examines it for possible violations of the first four sections, and the 6 Section of the Act. If the majority of prove it, the tariff is accepted.
    2. The Commission refers it to the standing rate committees of the carriers for determination of the lawfulness of the rates contained therein.
    3. The tariff is received by the Terrace Bureau of the Commission, and checked by its rate clerks for conformance to the provisions of the sixth Section of the Act. If conforming thereto, it is accepted and is permitted to become effective.
    4. The tariff is returned to the carriers with the statement, that since the burden of proof rests upon the carriers to justify the new rates, they must prove that the rates are lawful under the Act before the tariff can be allowed to become effective.
  3. (10 points)
    An ocean steamship line quotes a rate of $10 W/M on automobiles, New York to Liverpool. What would be the ocean freight on an automobile so shipped, weighing 4,000 pounds boxed, and measuring 120 in. by 60 in. by 50 in.?

Source:  University of Chicago Archives. Department of Economics, Records. Box 39, Folder 28.

Image Source: Element from the Social Science Research Building. University of Chicago Photographic Archive, apf2-07449, Special Collections Research Center, University of Chicago Library.

Categories
Chicago Curriculum Iowa Statistics

Chicago. Henry Simons argues for an undergraduate sequence of mathematics/statistics for economists, 1937

 

 

The letter below written by Henry Simons to Henry Schultz in 1937 is evidently a typed copy of what was originally a letter on official University of Chicago stationary. The typed header matches the printed header of University of Chicago stationary and there is no signature at the end.

Simons appears to be seeking Schultz’s support for the introduction of a “Mathematics for Economists” course into the undergraduate economics curriculum as well as for providing different courses for students who intend to go on to more advanced economics training versus the sort of survey courses that would constitute the entirety of the life-time economics education of non-econ-majors. An interesting aside: Simons problematized the lack of analytical preparation displayed by the students coming from Social Services Administration that he saw reducing the standards in the economics courses that they were required by their program to attend.

_____________________________

The University of Chicago
The Department of Economics

Memorandum to Members of the Department from Henry Schultz. July 8, 1937

The attached letter from Mr. Henry C. Simons might very well serve as a basis of discussion. It may be necessary to call a meeting to discuss this question before the quarter is over.

*  * *  *  *  *

THE UNIVERSITY OF CHICAGO
Department of Economics

June 4, 1937

Dear Mr. Schultz:

Out at Ames last week I heard about some plans of their economics department which made me very envious. They are getting ready to offer next year a sequence of three courses combining elementary mathematics and statistics; and they expect afterwards to make these courses prerequisites to their advanced (divisional) courses in economics. Moreover, they seem to be facing squarely the task (1) of providing a significant amount of training in the calculus, (2) of eliminating or cutting down those parts of the usual freshman mathematics which are of little use for their students, and (3) of mixing in with the formal mathematics perhaps as much statistics as is covered in a one-Quarter course.

If they carry out these plans, their students will soon be better prepared for substantial economics training than are even those few students here who complete Math. 104, 105, and 106—not to mention those who meet only our minimum requirement of one course in college mathematics. Meantime, nothing is being done to improve our situation here. Mrs. Logdon’s courses were a slight improvement over the old elementary mathematics; but they represent only a small beginning toward what might be done. The 104 course has its merits; but the two following courses, I gather, largely compensate for any departures from tradition which the first course involves. We still have not faced the fact that the traditional freshman mathematics, however suitable for students who will specialize in mathematics or physics, is very ill-suited to the needs of students going into other divisions or of those concluding their formal education at the college level.

I feel that we should face now the responsibility of providing a suitable minimum of training in mathematics, formal and applied, for all students in the College. Nothing can more easily be defended as a part of general education or as intellectual preparation for serious work in the Divisions. The need here might well be met by a three-course sequence of the kind which they are planning at Ames—although I am not competent to prescribe, or disposed to quarrel, about details. There are obvious advantages in mixing a certain amount of applied mathematics with the more formal training; and the fundamentals of statistics can be taught to best advantage only as mathematics and in the atmosphere of mathematics courses. As regards these fundamentals, there is no need for differentiation of courses according to divisions or departments—except possibly in the case of the physical sciences. With appropriate work in the College, divisional statistics courses in the various departments might then achieve their proper emphasis upon special applications in the special fields.

Our own Division probably could not now be induced to impose such a requirement for admission. Some departments would doubtless oppose it vigorously. This situation, however, does not argue against developing in the College the sequence of courses which would be most useful. If the proper courses were available, we could make them prerequisite for divisional work in economics; and, at the least, we could urge the advisers in the College to explain that students coming to us without such preparation would be somewhat handicapped on that account. Some other departments and divisions might go along with us. The Division of Biology certainly should do so; the School of Business and the Law School would probably cooperate eventually; and the School of Social Services Administration needs this sort of thing badly, both to protect their own standards and to guard us against the demoralization of standards which a large influx of their ill-prepared students can produce in the economics courses which they require.

It remains to point out that an important step could be taken now by our own department. Our announcements indicate that “Social Science II or equivalent” is prerequisite for divisional courses in economics. The policy here involved is, I believe, grossly mistaken. Instead of requiring this sequence, we should recommend against it in the case of students preparing for divisional work with us, or, at least, indicate clearly that the existing mathematics sequence is distinctly preferable as preparation. The typical student now gets a survey of social science in the first year, another in the second year, and still another (the five 201 courses) in the first two quarters of the third year. This represents an outrageous squandering of the student’s time, considering the alternatives actually sacrificed. Social Science II has perhaps a proper function; but it is not that of preparing students for divisional work. It may be appropriate to offer such a sequence for students who will enter other divisions and who will have no further work in social science fields. Our own students, however, should be getting more fundamental education—should be taking courses involving the more rigorous intellectual discipline in which their subsequent training will be somewhat  deficient.

If there be disagreement on some of these suggestions, there should be little opposition to my minimum proposal, namely, that Math. 106 be indicated in our announcements as a prerequisite alternative to Social Science II. Frankly, this is what it is in fact now, when I am acting as departmental counselor.

In passing, I will mention another suggestion which I have urged repeatedly in meetings and in memoranda. Something should be done to stop this concentrating of the 201 courses in the first two quarters of the student’s divisional work. These courses should not constitute merely another hurdle which students must get over before they are permitted to concentrate upon departmental courses. They should be spread throughout the last two years, as a continuing correction against narrow departmental specialization in outlook and interest—not studied hastily in a lump and forgotten.

The advisers in the College have finally discovered that Math. 104 is useful for students going into economics. They should all be told now to recommend105 and 106 as well and to suggest that good and serious students should be prepared to take at least some calculus after they leave the College (if not before). It is surprising how many of our seniors now complain bitterly because their College advisers failed to offer such suggestions.

I trust that some of these suggestions will seem to merit discussion.

Sincerely,

Henry C. Simons

 

Source: The University of Chicago Archives. Department of Economics. Records. Box 41, Folder 12.

Image Source: Henry Calvert Simons portrait at the University of Chicago Photographic Archive, apf1-07613, Special Collections Research Center, University of Chicago Library.
Henry Schultz from “[Photograph]: Henry Schultz 1893-1938.” Econometrica 7, no. 2 (1939).

Categories
Chicago Problem Sets

Chicago. Henry Simons’ classic problem set, 1933.

 

According to Martin Bronfenbrenner, the following problem set devised by Henry Simons for Chicago undergraduates in 1933 was a pedagogical Meisterstück (ok, he just said “one of the most famous problems in economic pedagogy”). It is likely that Paul Samuelson, who considered Simons his best teacher at Chicago, cut his teeth on this problem set as well.

________________

Economics 65-165

M. Bronfenbrenner

A General Problem in Competitive Price

This problem was originally devised by the late Professor Henry C. Simons for Chicago undergraduate classes in 1933. It has lived on to become one of the most famous problems in economic pedagogy. Give yourself plenty of time to work with it. It is not only long but abounds in pitfalls.

There are 1000 firms in a highly competitive industry which produces a standardized product. Each firm owns and operates one plant, which is of the most efficient size. All firms have identical costs, as follows:

Output per week

Total Cost

Output per week

Total Cost

Fixed

Variable

Fixed

Variable

1

$100 $10 13 $100 $101
2 100 19 14 100

113

3

100 27 15 100 126
4 100 34 16 100

140

5

100 40 17 100 155
6 100 45 18 100

171

7

100 50 19 100 188
8 100 56 20 100

206

9

100 63 21 100 225
10 100 71 22 100

245

11

100 80 23 100 266
12 100 90 24 100

288

The demand curve for the industry is given by: pq = $255,000. Your first task is to make out a demand schedule, and incorporate it in your solution as Appendix 1.

Part i

Draw the supply curve (the sum of the marginal cost curves) and the demand curve of the industry on the same graph (Fig. 1). Read off the equilibrium price and quantity. Prove that your answer is correct by comparing quantities supplied and demanded at prices $1.00 higher and $1.00 lower.

Draw the cost and demand curves of the individual firm on the same graph (Fig. 2). Accompany both graphs (Fig. 1-2) with textual explanation of their construction and of any differences between them.

 

Part ii

Congress unexpectedly imposes a tax of $4.00 per unit on the manufacture of this commodity. The tax becomes effective immediately and remains in effect indefinitely. Assume:

a. No changes in the economic system other than those attributable to the tax.
b. No change due to the tax has any effect on the prices of productive services used by the industry. (This assumption will be dropped later.)

  1. Draw the new supply curve and the demand curve of the industry (Fig. 3). Read off the new equilibrium price.
  2. Draw the new cost curves and the demand curve of the individual firm (Fig. 4). Explain the construction of these graphs (Fig. 3-4).
  3. Why can the price not remain as low as $15?
  4. Why can the price not rise to and remain at $19?
  5. Precisely what would happen if the price remained for a time at $16?
  6. At precisely what level would the price become temporarily stable? What does it mean to say that this is an equilibrium level?
  7. Suppose the short-run equilibrium price to be $17. How would you answer the query:

“I don’t see why every firm should produce 15 units per day when the price is $17. It would make just as much if it produced only 14, for the 15thunit adds just as much to expenses as it adds to revenues.” Precisely what would happen if some firms produced 14 units per day and others 15 units?

  1. Would short-run equilibrium be reached at a higher or lower price (and with larger or smaller output) if the elasticity of demand were lower (less than unity? If it were higher (greater than unity)?
  2. What would happen if demand had an elasticity of zero? An elasticity of infinity?

 

Part iii

As Figure 4 will reveal, the new minimum average cost is $19. The short-run equilibrium price was $17; hence this industry becomes unattractive as an investment, relative to other industries. As plants are worn out, therefore, they will not be replaced; plants will be junked sooner; and even maintenance will be reduced. To simplify the problem, we assume:

  1. Each plant has a life of 1,000 weeks.
  2. The plants in the industry are staggered so that, at the time the tax was imposed, there is one plant 1 week old, one plant 2 weeks old, etc.
  3. At the time the tax was imposed, 20 plants were so near completion that it is impossible to divert them to other uses. These are completed at one-week intervals.

Hence for 20 weeks the price will stay at $17, and then rise gradually as entrepreneurs fail to replace worn-out plants.

  1. What will the situation be at the end of the 25thweek? (Answer in terms of “greater than” or “less than.”)
  2. When 120 weeks have passed (900 plants left), will the price be above or below $18? Explain carefully.
  3. How many weeks must pass (how many plants must be scrapped) before the price rises to $18? Explain precisely.
  4. Will the output per plant increase or decrease as the number of plants declines?
  5. When 220 weeks have passed (800 plants left), will the price be above or below $19?
  6. How many plants must be scrapped before the price rises precisely to $19?
  7. What would the price be if the number of plants declined to 750? What would be the output per plant? What would happen to the number of plants?
  8. What happens to the short-run supply curve of the industry as the number of plants diminishes? Draw, on the same graph (Figure 5), the supply curve when there are 1,000 firms and 800 firms. Compute elasticities of supply for these two curves at a given price.
  9. How could the process of adjustment, and the final equilibrium, be different.
    1. If the elasticity of demand were greater than unity?
    2. If the elasticity of demand were less than unity?
      (The significant points are: (1) price, (2) output per plant immediately after the tax is imposed, and (3) number of plants and total output at the new long-run equilibrium).

 

Part iv (Optional)

Finally, the prices of the productive services will be affected by the purchases of the industry. Some of the services will be specialized: Larger quantities can be secured only at higher prices, and smaller quantities can be secured at lower prices. Assume that all of these services are “fixed”, and that all variable services are unspecialized (i.e., any quantity can be secured by the industry at a constant price).

  1. Will the short-run effects of the tax be any different than they were in Part 2? Explain in detail.
  2. How will the long-run adjustment differ? Will the final price be more or less than $19, and the daily output more or less than 13,421? Again explain in detail.
  3. Suppose that a special and scarce kind of land is required for production of the taxed commodity, and that this land is not used (or within practicable limits usable at all) in the production of any other commodity, and that all other resources are completely unspecialized. What is likely to be the effect of the tax on the price of the use of such land (on its rent)?
  4. Suppose that this special and scarce land is also used in one other industry. Will the rent of this land fall more or less, if the demand for the product of this second industry is elastic or inelastic?

 

Source:   Duke University. David M. Rubenstein Rare Book and Manuscript Library. Economists’ Papers Archives. Papers of Martin Bronfenbrenner, Box 26, Folder “Micro-econ & Distribution, 1958-67, n.d. 2 of 2”.

Image Source:  Henry Calvert Simons portrait at the University of Chicago Photographic Archive, apf1-07613, Special Collections Research Center, University of Chicago Library.

 

 

Categories
Chicago Funny Business M.I.T.

M.I.T. Christmas skit “God and Keynes at M.I.T.”, 1951

 

The title of the Christmas skit presented by the Graduate Economic Association players at MI.T. in December 1951 , “God and Keynes at M.I.T”, is a clear reference to the political screed, God and Man at Yale (1951), by the young and future conservative pundit, William F. Buckley, Jr. This is one of many MIT skits found in the papers of Robert M. Solow and has been graciously shared for ERVM transcription by Roger E. Backhouse of, most recently, Becoming Samuelson, 1915-1948 fame.

One of the signs you are dealing with truly academic humor is the use of footnotes to provide proper attribution. In particular we find here seven items borrowed (and sometimes modified) from the University of Chicago Political Economy Club repertoire. Thus we see not only were some of the Greatest-Hits of Chicago skit humor “remastered” in the Windy City but also that the G.E.A. of M.I.T. was not above performing “covers” of Freshwater Hits. ERVM has already transcribed a few of these and for the sake of completeness will soon complete this list with the Chicago originals:

There is still plenty of original material in the following skit, and the few modifications worth noting include a key substitution of Keynes (MIT) for Marshall (Chicago)  and another substitution of “psychology and sociology” (MIT) for “Macroeconomics and Probability” (Chicago).

________________________

THE GRADUATE ECONOMICS ASSOCIATION
present
The G. E. A. Players
in
GOD AND KEYNES AT M. I. T.
15 December 1951

*Items so marked are modified versions borrowed from the University of Chicago, Political Economy Club.

 

 

PROLOGUE

(the scene is set to reveal the young college graduate relaxing in his home. He has made application to M.I.T. for entry to Course XIV. We hear the door-bell ring, and the letter arrives. He reads:)

An economics department great in dignity
In fairest Cambridge, where we lay our scene
Offers to disturb you, from present peace
To come to our proximity.

From forth of this great and new transition
A host of new subjects will take their position;
Econometrics, propensities, and laboristic relations;
Matrices, consumption, and similar sensations.

And if you will survive the economic pains
We’ll make of you another John Maynard Keynes.
So won’t you please say that you will come and stay;
Let me know real soon, signed sincerely, C. P. K.

(the student arrives at Tech, finds the library, and enters the elevator. On the way up to the third floor he hears:)

 

FIRST EPISTLE UNTO NEW STUDENTS*

  1. To all who enter through the Gate of Admissions unto the sanctity of the Department, heed ye well one who is wiser and older than thou. For verily I have dwelt in the land of Keynes for many years, and have felt the curse of Generals on my brain.
  2. Beware the courses called 121 and 122, for they will tax thee sorely. They have been devised that the supply may be known from the demand.
  3. Present thyself upon the appointed hour, lest the social cost exceed the private gain and the wrath of the Master fall upon thee mightily.
  4. Shun thou the geometer, for he seeks to seduce thee with curves. His siren song is pleasant but he lacketh rigor.
  5. Shun thou also the temple of the twin gods, psychology and sociology, for therein dwell the Philistines who worship not the calculus. There wilt thou be set upon with all manner of strange things and thou shalt feel the lash of the complex verbage, and thy head shall whirl with cultural patterns and institutional mores.
  6. Treasure thy Keynes, for verily all manner of mysteries are set down therein. Read it well and carefully, but say not that thou hast understood.
  7. Take to thine own bosom the demand curve lest it desert thee in thine hour of need.
  8. Attend well the lectures called innovation, for there if thou learnest nothing else, shalt thou learn at least one thing and it shall be a contribution to thy general education.
  9. Shun thou the industrial economist when he is at his data, for he loveth them dearly and will defend them as a lioness her cubs.
  10. Beware also the statistician who will leave the witless with a pair of dice.
  11. Shun the welfare economist, for he loveth mightily to stick out his neck and will teach thee his evil ways.
  12. Shun thou the coffee hour, but study diligently in Dewey lest thou and thy end thy days in Course XV.
  13. There is a time to speak and a time to be silent. Be thou silent in the presence of the Master, for he shall reveal to thee the secrets of Keynes and there shalt thou solve the riddle of the Sphinx.

 

(the student steps out of the elevator into the third floor hall. He sees before him many doors, all with different names on them. He decides to investigate each one. First, he comes to:)

“John Maynard Keynes”

(he knocks. The door opens, and out steps an angel, wings, white sheet, and all. The angel says:)

‘He ain’t here; but you’ll meet him in the long run!’

(on to the next door:)

“Paul A. Samuelson”

(the door opens, and the chorus sings:)

THE KEYNESIAN SONG*
(to the tune “They Call me Little Buttercup”)

They call me a Keynesian, a Keynesian economist
That I can never deny
For I am a heretic, a classicist critic—
Bold little Keynesian, I.

I’ve equations and functions, and marginal assumptions
All here in my little kit bag.
I’ve tricky proposals for income disposals
All lest the economy sag.

To deficit spending and government lending
I give a hearty “Huzzah”.
I distrust automaticity despite its simplicity—
I doubt it would work at all.

For I am a Keynesian, a Keynesian economist
That I can never deny
For I’m a heretic, a classical critic—
Bold little Keynesian, I.

When faced with deflation or misallocation
I feel that the former is worse
I abominate waste with Ricardian distaste
But first things always come first.

And yet they deplore me, criticize and abhor me
For I am the standard straw man
But blows I don’t heed—Oh, I’ll stick to my credo
That a plan is a plan is a plan.

For I am a Keynesian, a Keynesian economist
That I can never deny
For I’m a heretic, a classical critic—
Bold little Keynesian, I.

 

“Robert Solow”

(scene, his classroom, where the students are singing:)

 

WE MUST BE RIGOROUS*
(to the tune of “The American Patrol”)

We must be rigorous,
We must be rigorous,
We must fulfill our role;
If we hesitate
Or equivocate,
We won’t achieve our goal.
We must investigate
Our system, complicated
To make our models whole;
Econometrics brings about
Statistical control.

Our esoteric seminars
Bring statisticians by the score.
But try to find economists
Who don’t think algebra a chore.
O, we must urge them all emphatically
To become inclined mathematically
So that all that we’ve developed, may
Someday be applied.

(repeat first 11 lines)

 

 

“Charles P. Kindleberger”

(the door opens, and we hear a voice say:)

Intuition is the basis
on which decisions should be made;
These are really the foundations
On which economics has been laid.

All that’s mathematical
Definitely is tabled;
Even the little diagrams
Never have been labeled.

Be careful, however
That you never neglect
The varied use
Of the Kindleberger effect.

Art or skill
or merely a quirk
This man’s intuition
Does the work.

 

 

“Robert L. Bishop”

(the door opens, and we find snow falling. The chorus is on a toboggan, singing:)

(to the tune of Jingle Bells)*

Maximize, maximize, that’s the crucial key;
Allocate resources by their productivity.
Equalize V.M.P.’s with their prices, and
Your production function is the finest in the land.

 

(voice) In the course of industrialization men have observed the alternating rises and falls of economic activity. And, lo, see what befell us:

“Walt W. Rostow”

(the voice continues:)

To shoot, or overshoot, ah, there’s the cycle;
Whether ‘tis nobler from underinvestment to suffer
Than to prolong the period of gestation
And, by consumption end it?

To history! No more of economics; and by the use of it
To end the confusion and million little theories
That economics left us;
That’s the solution we plan to introduce.

 

“E. Cary Brown”
(to the tune of “Deep in the Heart of Texas”)

(chorus)

To fill the gap
On the Keynesian map
We must again raise taxes;
The prices rise
If we don’t equalize
Savings, investment and taxes.

(solo)

Income grows
In ever rising flows
We must again raise taxes;
In government spends
There seem no ends
Up must go the taxes.

(solo)

dC/dY
Is all awry
We must raise those taxes
The propensity
It’s a calamity
Up must go those taxes.

(chorus)

The interest rate
Is out of date
So we must raise those taxes;
Though bonds recede
We must proceed
To raise again those taxes.

(solo)

The crystal balls
In the third floor halls
Say raise those taxes;
Or you will fret
And long regret
If you don’t raise those taxes.

(solo: and how!)

Flexibility
Cries the C.E.D.
Boys, raise those taxes
Says the N.A.M.
It’s all a sham
Don’t raise those taxes

(chorus)

But God and Keynes
Have the true refrains
Up must go the taxes;
At M.I.T.
We all agree
More savings and more taxes.

(by now, our student has traveled one-half the length of the hall. He approaches the other half, where a voice speaks:)

 

Friend; first year man; lend me your ear.
I come to convince you that industrial relations
Occupies a so much higher station
That economics—while ’t is good and fine
Must of necessity bow under our sign.
The evil that me do lives after them;
The good is oft interred within their books;
So let it be with economics.

We offer to show you the extent of cooperation
Between management and labor in every relation,
And prove to you that what’er your belief
Our unique methods will give either side full relief.

Economists, you know, often speak of productivity;
But that’s a matter of total relativity
Since our writers—Shultz, Myers, Coleman and Brown
Are the most productive in a many a college town.

 

“Charlie Myers”

(the door opens, and we see Myers writing vigorously and adding stacks of manuscripts to already huge piles labeled “To Prentice Hall,” “To McGraw-Hill,” and “Rejects—to Technology Press.” Secretary enters:)

Secretary: “Prof. Myers, here’s that book you asked me to write for you.”

Myers: “Good; don’t forget to start on that other one for me.”

(enter George Shultz carrying a manuscript)

Myers: “Hello, George. I see we’ve written another book. Mind if I look at it?”

Shultz: “Not at all, Charlie. I’ve already begun on the other one for us. You know, though, I think we’re getting a bit too abstract. We ought to go down to a level where it’s good and dirty.”

Myers: “In that case, let’s call in Joe Scanlon. Hey, Joe. Come here.”

(the chorus enters, dressed as bums; they sing:)

THE JOE SCANLON SONG
(to the tune of “Union Maid”)

There once was a bright young man
Who thought he had a plan
He studied cost
And jobs he lost
His name is Joe Scanlan

He soon met a man named Phil
Whose work gave him a thrill
He organized and compromised
He always fought up-hill.

This made of him a wreck
And so he came to Tech.
He sells his plan
To all the clan;
You ought to see his check.

CHORUS:
O you can’t scare us, we’re sticking with Scanlon,
Sticking with Scanlon, sticking with Scanlon;
Oh you can’t scare us, we’re sticking with Scanlon,
Sticking with Scanlon, until we die.

 

When the bosses have no dough
They always call for Joe;
They shed their tears
And buy him beers
And up their profits go—

(repeat CHORUS)

 

(as the final chorus ends, the door opens, and we see a body on the table)

Bishop: “What’s the matter with him, Morrie Adelman?”

Adelman: “He’s just been brought in; he’s suffering from a severe case of elephantiasis.”

Bishop: “Oh, don’t worry; I’ve got a classical solution. It contains some of Euler’s serum.” (pull up a jug so labeled and apply to patient’s arm)

Adelman: “Well, what do you expect that to accomplish?”

Bishop: “It’ll create perfect competition among the disease germs. What could be better?”

Adelman: (pause) “Well, I don’t see him recovering.”

Bishop: “But it’s not a pure case. Perhaps we should call in Dr. D. V. Brown. He’s had medical experience. (enter D.V.B.)

Brown: “Hi-ja.” (looks at body, and shows surprise) “My goodness, Charlie! I always knew he’s work too hard.” (looks at body more closely) “Looks to me like an impure case of oligopoly.”

Adelman: “O-o-o-oh! Let me see!” (goes over to feel arm) “No, there’s no concentration here. But even if there were, there’s really no harm in it.”

Brown: “Well, I’d like to stay, but I have to dash off to a court case.”

 

COURT SCENE

Judge: “The court is now in session. Bring in the first case.”

Prosecutor: “Your honor, this man is accused of attempting to overthrow the neo-classical Chicago School.”

Judge: “What’s your name?”

Coleman: “Sir, my name is Jack Coleman.”

Judge: “Prosecutor, define more explicitly what the charge is against this man.”

Prosecutor: “This man is presently collaborating with a well-known group of collectivists.”

Judge: “What proof have you of this?”

Prosecutor: “I have here my star witness.”

Judge: “What is your name?”

Buckley: “Your honor, sir, my name is Ludwig von Buckley.”

Judge: “Speak.”

Buckley: “I have here a book written by Paul A. Samuelson, and it says here on page.–., Oh, well, let’s not bother with the page number now. It says: “…know…conclusively…that…Karl Marx…is…(turn pages back towards front)…correct.”

Judge: “Speak no more. Any man collaborating with the author of such a book must be guilty of attempting to overthrow the Chicago School. I hereby sentence you to six months of solitary confinement, with a copy of Hazlitt’s “Economics in One Lesson.” Next case.”

(Coleman leaves; enter Herb Shepard)

Prosecutor: “Your honor, this man is accused of playing marbles with the fabulous Alex Bavelas.”

Judge: “What is your name?” (say it aggressively)

Shepard: “Say, you’re unusually aggressive today. Has your wife stopped beating you? How’s your libido?”

Judge: “Now that you mention it, I have been feeling rather despondent.”

Shepard: “Judge, I’m a Freud…you’re tending toward a psycho-social orientation that no longer promotes an optimization of gratification.”

Judge: “Noooooo—I’m too JUNG to die!….But what am I saying! Herbert Shepard, for this circumlocutionist behavior, I hereby sentence you to the marble pits in ex-communication.”

 

(the student next comes to a door marked “reserved for Chicago U. delegates to the A.E.A. Convention.” He knocks, the door opens, and he hears:)

 

HIS RULES GO MARCHING ON*
(to the tune of the Battle Hymn of Republic)

If you want to pass your prelims
You must listen now to me;
You must learn your catechism
If you want to get your ‘B’
They have flunked the finest people
The department ever had
And they never said ‘too bad.’

CHORUS:

Stick, stick, stick with Henry Simons;
Henry is the man to see you through;
He’s the most consistent [man]
With an economic plan;
His rules go marching on.

 

He would nationalize the railroads,
He would atomize the firm,
He would then repeal the tariff
And the “E” bonds he would burn;
He would cleanse the banking system
Of the Federal Reserve;
His rules go marching on.

[Repeat] CHORUS:

He is the man who’d fix up
The progressive income tax;
He would fill in every item that
The present structure lacks;
He’d repeal the excise levies
And forget the margarine tax;
His rules go marching on.

[Repeat] CHORUS:

 

(by now the student will have reached the end of the hall; but questions linger in his mind. He wonders how the student takes all this. And as if in answer, he hears this song between students and faculty:* (to the tune of the ‘Sergeant’s Song’ from the Pirate[s] of Penzance)

Grad Students:

From nine around to nine—Tarantara! tarantara!
We remain in that salt mine—Tarantara!
-Our eyes are growing dim–Tarantara! tarantara!
Our hair is getting thin—Tarantara!
As we while away our youth—Tarantara! tarantara!
In sedate pursuit of Truth—Tarantara!!
Searching stacks and aching backs,
Third degree for a PhD—Tarantara! tarantara! tarantara!

 

Faculty: (to the tune of “Mabel’s Song” from the Pirate[s] of Penzance)

Go, you students, you’ll not be sorry.
You’ll contribute to MY great story.
You shall live in footnote glory.
Go to immortality!

Go to work and hold off suicide,
For if your work with our needs coincide,
Our reluctance to grant degrees we’ll override.
Go, you heroes, go and work!

 

(finally, as our student reaches the end of his journey, he meet the one ‘older and wiser than thou’, and listens as he tells of the ‘impending doom’.)

Twas the night before Orals
When all through the room
A feeling forecast
The impending doom.
The facts were placed
In each head with care
In hopes that when needed
They’d surely be there.
The victims then nestled
All snug in their beds
While visions of cost curves
Danced in their heads.
I soon fell asleep
And began to dream
I sat in a room
All filled with steam.
When out in the yard
There arose such a clatter
I sprang from the chair
To see what was the matter.
Over to the window
I flew like a flash
Tore open the shutters
And threw up the sash.
When what to my wondering
Eyes there appears
A miniature sleigh
And eight tiny examineers.
Instead of the four
They usually required
They sent me four more
If the others got tired.
As I drew in my head
And was turning around
In through the window
They came with a bound.
They were dressed all in black
From their head to the toe;
Whose funeral, I asked,
Someone I know?
A wink of their eyes,
A twist of each head
Soon gave me to know
I had plenty to dread.
They spoke not a word
But went straight to their work
Of filling the blackboards
Then turned to the jerk.
The questions commenced
Like machine gun fire;
I couldn’t keep straight
The seller from buyer.
Now sir, please listen
One of them said
Try to imagine
All this in your head.
Nansen and Johansen
Have only one sled;
They’re at the North pole
And have not bread.
Suddenly there appears
A giant Tartar
Coming from Siberia
Looking to barter.
They can bake some bread
At increasing cost
Yet without a compass
They’ll certainly be lost.
He has a compass
And they have bread
And without exchange
They all will be dead.
They started to bargain
Until he did tell you
That the Russians decided
The ruble to devalue.
Only Sterling is recognized,
So they start to bake
Instead of the bread
A large pound cake.
Then suddenly Nansen
Thought to remember
That neither of them
Was a union member.
Closed shops were enforceable
As a matter of fact
For this was before
The Taft-Hartley Act.
They went ahead anyway,
They didn’t give a hoot;
It was so cold
They needed a union suit.
Before they acted
Or did anything drastic
They examined their demand curve
To see if it was elastic.
Their cost curve was unknown–
It had never been seen;
How lucky they were
That Nansen was really Joel Dean.
Their consumption function told them
Just how to behave;
They knew what to consume
And how much to save.
Please consider the theories
of Tibor Scitovsky
And the two fisted cowboy
two-gun Baranowsky.
If you remember these facts
And keep them in mind,
The right answer, I know
You certainly should find.
I shivered and shook,
In the chair I did writhe;
Now the question, they said
Who was Adam Smythe?
The leader then yelled
For a decision it’s time;
This man has suffered,
He has paid for his crime.
And laying a finger
Aside of his nose
Out of the window
All eight of them goes.
It was the leader then
That I heard exclaim
As he shouted and whistled,
And called them by name:
Now Myers, now Bishop
Now Shultz and C.P.K.
On Coleman, on Solow,
Let’s now dash and dash away.
They sprang to their sleigh
And away they flew
Like they were speeding
To another rendezvous.
Although some details
Of this horrible nightmare
Still seem a bit hazy
I certainly would swear,
Before I awoke
I heard them say
Merry Christmas to all,
And to all a good day.

 

EPILOGUE

As disproved by classical economics
All good things much reach an end;
And so we must leave our attempt at comics,
Hoping we’ve pleased both foe and friend.

‘Tis true enough that our little parody
Has given economics unusual clarity,
And that our writers if circumstances permit it
Will prefer to have their names omitted.

So then, since ours must be the last say,
a real Merry Christmas from the G.E.A.

 

Source: Duke University. David M. Rubenstein Rare Book & Manuscript Library. Economists’ Papers Archive. Robert M. Solow Papers, Box 83, Folders “Economic Skit Parties”.

Image: Cover art from “God and Keynes at M.I.T.” December 15, 1951. Ibid.

 

Categories
Chicago Courses Suggested Reading Syllabus

Chicago. Henry Simons’ last course. Fiscal Policy, 1946

 

 

Henry Simons’ course “Economics of Fiscal Policy” was introduced into the Chicago public finance offerings in the Winter Quarter of 1934-35 and was taught by him in all but two years before his suicide that happened immediately after the Spring Quarter of 1946 had concluded.

From Norman M. Kaplan’s student notes for Simon’s last course I have transcribed the list of course readings and the rough outline of the course discussed in the first two sessions.

________________________

Course Announcement

[Economics] 361. Economics of Fiscal Policy.—A study of fiscal practices with reference to (1) booms and depressions (budget-balancing), (2) distribution of income (inequality), and (3) composition of the national income (incidence). The latter weeks will be devoted to study of particular kinds of taxes, especial attention being given to problems of income taxation. Prerequisite: Economics 209 and 230 or equivalent. Spring: MWF 11; (joint meetings with Law 510e for a part of the Quarter, additional hours to be arranged); Simons.

Source: University of Chicago. Announcements: The College and the Divisions, Sessions of 1945-1946. Vol. XLV, No. 7 (June 15, 1945), p. 219.

________________________

 

Readings and Course Outline According to Kaplan Notes

March 27 [1946]

  1. Readings:

Hansen & Perloff, State and Local Finance in the National Economy (preferred). Chs. 9, 10, 11, 12 first, then parts I & II.

or Hansen, Fiscal Policy and Business Cycles

McGill, The Impact of Federal Tax (Latter part of course)

Simons, Personal Income Taxation (Latter part of course)

“Rules vs. Authority in Monetary Policy”, Simons

“On Debt Policy”, Simons

“The Beveridge Program, an Unsympathetic Interpretation” [optional], Simons

Public Finance and Full Employment, Fed. Res. Board publication on series of post-war studies. (Important for 1st part of course, especially 1st 2 papers, then Robinson and final paper.)

Part II of Groves, Financing Government, for those using this as a survey course.

Read portions of Beveridge book which have to do with fiscal policy, part I, part IV, appendices B & C. esp. sec. 4 of B and Append. C.

 

March 29 [1946]

  1. Topical Sequence of course

A. First part: monetary fiscal budgetary policy (Last part of Hansen, Fed. Res. Bd., Rules vs. Authority, Beveridge stuff[?])

B. Then justice, incidence, etc. of taxation

1. Justice of taxation Ch. 1 of Simons Personal Income Taxation 

2. Incidence of taxation Brown, Econ. of Taxation

 

Source: University of Chicago Archives. Norman M. Kaplan Papers, Box 1, Folder 6.

Image Source: University of Chicago Photographic Archive, apf1-07613, Special Collections Research Center, University of Chicago Library.

Categories
Chicago Economists

Chicago. Simons urges the recruitment of Milton Friedman, 1945

 

 

The atomic bomb dropped on Nagasaki was less than two weeks history and the declaration of the surrender of Imperial Japan only five days old. Nothing says “back to business as usual” at the university better than active lobbying on behalf of one’s preferred candidate for an upcoming vacancy, as we see in the following memo for the 33 year old Milton Friedman written by Henry C. Simons to the Chicago economics department chair, Simeon E. Leland. The copy of this memo comes from the President’s Office at the University of Chicago. Simons’ grand strategy was to seamlessly replace the triad Lange-Knight-Mints with his own dream team of Friedman-Stigler-Hart. He feared that outsiders to the department might be tempted to appoint some convex combination of New Dealer Rexford Tugwell and trust-bustin’ George W. Stocking Sr., economists of the institutional persuasion who were swimming on the edges of the mainstream of the time.

Economics in the Rear-view Mirror also has transcribed excerpts from an earlier 77 page (!) memorandum (10 April, 1945) to President Robert M. Hutchins from Simeon E. Leland entitled “Postwar Plans of the Department of Economics–A Wide Variety of Observations and Suggestions All Intended To Be Helpful in Improving the State of the University”.

____________________________

 

Henry C. Simons Urges his Department Chair to Recruit Milton Friedman

August 20, 1945

To: Simeon E. Leland           Economics

From: Henry C. Simons        Economics

 

If Lange is leaving, we should go after Milton Friedman immediately.

It is a hard choice between Friedman and Stigler. We should tell the administration that we want them both (they would work together excellently, each improving what the other did), Friedman to replace Lange, Stigler to replace Knight and to be with us well ahead of Knight’s retirement. We might also say that we want Hart to replace Mints at Mints’s retirement, and also to be with us in advance, but are happy to have him financed by C.E.D. [Committee for Economic Development] for the present.

Yntema evidently is thinking of getting Friedman shortly. We should exploit this possibility. Milton has now a great yen for a University post and would probably turn down an offer from C.E.D., even at much financial sacrifice, if a good academic post were the alternative (as it might be, at Minnesota or elsewhere). He is rather footloose—not anxious to go back either to the Treasury or to the National Bureau. We should grab him now, offering temporary joint appointment with C.E.D. and full-time, permanent appointment when he is through with C.E.D.

Friedman is young, flexible, and available potentially for a wide variety of assignments. He is a first-rate economic theorist, economic statistician, and mathematical economist, and is intensely interested over the whole range of economic policy. He has been outstanding in every organization where he has worked—here with Henry Schultz, at the National Bureau, at the Treasury, and now recently in the Army project at Columbia. Moreover, he is one of those rare cases of able young men who have enjoyed large experience and responsibility in Washington without being at all disqualified thereby for academic work.

The obvious long-term arrangement is a joint appointment with the Cowles Commission. Marschak would, I’m sure, like to have him; and Milton would like to settle into a major project of empirical research, e.g., on enterprise size and productional efficiency. Bartky may be expected strongly to support the appointment, for its strengthening of the University in statistics. The School of Business could well use Milton, to give its few advanced courses in statistics, if Yntema continues to price himself out of the University. Moreover, Milton probably would be delighted to work partly in the Law School, and be extremely useful there. In the Department, he would be available for statistics, mathematical economics, pure economic theory, taxation, and almost any field where we might need additional courses.

If University officers want outside testimony, they could get it from Randolph Paul or Roy Blough (as regards the Treasury), from Arthur F. Burns (National Bureau), from Abraham Wald, Allen Wallis, and Barky (as regards war research), and from Bunn at Wisconsin (as regards possible usefulness to the Law School)—not to mention George Stigler, Harold Groves, Wesley Mitchell, Simon Kuznets, Erwin Griswold, et al.

Perhaps the best thing about Milton, apart from his technical abilities, is his capacity for working as part of a team. He is the gregarious kind of intellectual, anxious to try out all his ideas on his colleagues and to have them reciprocate. He would doubtless be worth his whole salary, if he neither taught nor published, simply for his contribution to other people’s work and to the Department group as a whole. But he is also intensely interested in teaching, and far too industrious not to publish extensively. Our problem would be not that of finding ways to use him but that of keeping him from trying too many tasks and, especially, of leaving him enough time for his own research.

It would, I think, be good policy and good tactics to submit a major program of appointments, including [Frank W.] Fetter, Friedman, Stigler, Hart, and an economic historian (Innis or Hamilton), in the hope of getting them all within a few years, some on joint appointments with, notably, the Cowles Commission, the Law School, the School of Business (?) and, temporarily, the C.E.D. Research Staff. Such a program would serve to protect us against administration pressure for less good appointments (e.g.,  Stocking [George Ward Stocking, Sr., Ph.D. Columbia, 1925]), and from Hutchins’s alleged complaint that, while he wanted to consider major appointments in economics, the Department simply would not make recommendations. We should, in any case, err on the side of asking for more appointments than we can immediately get. Otherwise, available funds may go largely elsewhere—e.g., into Tugwell-like, lame-duck appointments, and into Industrial Relations, Agricultural Economics, and other ancillary enterprises, at the expense of the central field of economics.

There is, I trust, substantial agreement within the Department, on the men mentioned above. This fact, if fact it is, should be made unmistakably clear to the administration.

Incidentally, if we are going to explore possibilities of an appointment in American economic history (and I’m probably alone in opposing), we should do so only in co-operation with the History Department and with (from the outset) joint plans for joint appointments.

 

HCS-w

 

Source: University of Chicago Archives. Office of the President. Hutchins Administration. Records. Box 73, Folder “Economics Dept., 1943-45”.

Image Source: University of Chicago Photographic Archive, apf1-07613, Special Collections Research Center, University of Chicago Library.

Categories
Chicago Courses Problem Sets Suggested Reading Syllabus

Chicago. Intermediate Economic Theory for Non-Majors, ca. 1933

 

 

Today’s post is provides an undated reading list, a partial course outline and the preliminary motivating statement for an intermediate level undergraduate course in economic theory targeted to non-majors in the University of Chicago’s Division of Social Sciences. This material was found in a folder in George Stigler’s papers. He was a student at the University of Chicago from 1933-1936, but it is unlikely that he took this course. One presumes he acquired a copy on his own account then. As far as the authorship, I have not had time to compare this material with that of Henry C. Simons cited in the following bibliographic tip. However the style does appear to have Simons’ handwriting all over it. Kyrk and Mints also regularly taught this course during these years.

Bibliographic Tip:  Notes to Henry Calvert Simons’ Course Economics 201 (1933-34) taken by F. Taylor Ostrander and Helen Hiett were published in Research in the History of Economic Thought and Methodology. Volume 23, Part 2. Documents from F. Taylor Ostrander, Warren J. Samuels (ed.). Emerald, 2005.

_____________________

Course Description

201. The Divisional Course in Economics.—A survey of price and distribution, monetary, and cycle theory, developed chiefly through the use of a series of problems. The course is designed primarily to meet the needs of students who are majoring in departments other than Economics and who expect to take the Divsional Comprehensive Examination in Social Science. Prerequisite: Social Science I and II or equivalent, or consent of instructor.

Source: University of Chicago. Announcements, Arts, Literature and Science (for the sessions 1933-34), vol. XXXIII, March 25, 1933, No. 8, p. 265.

_____________________

ECONOMICS 201
MATERIALS AND PROBLEMS FOR CLASS DISCUSSION

ASSIGNMENTS

Indispensable Reading, first five weeks:

Henderson, H. D., Supply and Demand (New York: Harcourt, Brace and Co., 1922. $1.25).

This short treatise should provide a good review of previous work (it is among the materials for Social Science I). It should be read promptly, to renew acquaintance with the terminology and central propositions of economic theory; and the relevant chapters should be re-read later on, in connection with the class discussion of special topics.

Knight, F. H., in Syllabus and Selected Readings for Social Science II, pages 125-250.

This is also a review assignment; but no other material is likely to prove more valuable in connection with the first part of this course.

The first section (pages 125-137), on “Social Economic Organization and Its Five Primary Functions,” should be read promptly, in connection with the class discussion of the first week.

Ely, R. T. et al., Outlines of Economics, 5th ed. (New York, 1930), Chapters IX, X, XI, XX, and Appendix A. (The corresponding chapters in the 4th edition will serve equally well for this course.)

The first three of these chapters should be read as one assignment. The first part of Chapter XI deals with what are, from the point of view of this course, highly controversial questions. Chapter XX merits very careful study.

Gray, Alexander, The Development of Economic Doctrine (New York, 1931), Chapters III, V, and VI.

The chapters should acquaint students with the main ideas of the mercantilists, and of Hume, Adam Smith, Malthus, and Ricardo. Appendix A of the Ely book should be read in connection with this assignment.

Indispensable Reading, last five weeks:

Roberson, D. H., Money, new edition revised (new York: Harcourt, Brace and Co., 1929. $1.25).

This is an excellent, concise treatise by a leading English (Cambridge) economist. It should be studied with care, preferably in advance of class discussion of money and banking.

Ely, R. T. et al., Outlines of Economics, 5th ed., Chapters XIII to XVIII inclusive.

These chapters also merit careful, deliberate study.

Gregory, T. E., The Gold Standard and its Future, 2nd (or 1st) ed., London (and New York), 1932.

An unusually fine treatise, excellent for its fundamental analysis, and closely relevant to currently interesting and urgent problems.

Optional Reading:

Ely, R. T. et al., Outlines of Economics.

Gray, Alexander, The Development of Economic Doctrine.

Cassel, Gustav, Fundamental Thoughts on Economics.

Cassel, Gustav, The Theory of Social Economy (Barron translation), Book I and Book II.

Marshall, Alfred, Principles of Economics, 8th edition, especially Book V.

Hardy, Charles O., Credit Policies of the Federal Reserve System.

 

Preliminary

Economics 201: Its Place in the Curriculum:

This course is intended primarily for students preparing for the Divisional Examinations, and not for students majoring in the Department of Economics. It is designed for students who have had Social Science I and Social Science II in the College, and for those students transferring to the Social Science Division from other colleges who have had some previous work in economics. In general, the course will presuppose some familiarity with the terminology of economics and some ability to follow careful analysis.

General Description of Content of the Course:

The course falls, as to subject matter, into two main parts. The first six weeks will be devoted to study of “price theory”—to study of the forces governing, in an exchange economy, the determination of relative prices and the allocation of resources among different, alternative uses (assuming a money economy but disregarding, or abstracting from, monetary disturbances and cyclical fluctuations). This part of the course is designed to give students a critical understanding, first and above all, of how a competitive system works and, second, of how the introduction of monopoly in particular areas will affect relative prices and relative production. The latter part of the course will be devoted to study of money, banking, and business cycles—to study of factors governing the general level of prices and, more especially, to analysis of forces underlying the cumulative, self-aggravating maladjustments of booms and depressions.

The total quantity of required reading is intended to be moderate; and it is to be hoped that students will do this relatively small amount of reading with considerable care — with serious effort to comprehend thoroughly and to understand, rather than with the intention of accumulating information or memorizing propositions. If a student must choose between doing all the reading but doing it hastily, and doing a smaller amount with care, the latter procedure will prove decidedly more profitable. The assignments are designed, however, to eliminate the necessity of such a choice.

Most of the class hours will be devoted to discussion of specific problem-exercises designed to bring out, and to give precision to, the central concepts and propositions of price theory and monetary theory. Little effort will be made to relate the class discussions from day to day to particular parts of the assignments; but familiarity with the required readings will always be helpful, and sometimes indispensable, to understanding of problems dealt with in class.

A considerable part of the student’s outside work should be devoted to assimilating and organizing in his own mind the content of discussions in class. Students should make a special effort to acquire facility with the language of more rigorous economics — with the main terms and concepts —, to understand clearly the assumptions under which particular analytical arguments proceed, to digest the analysis of particular problems as it proceeds in class, and to prepare themselves to carry on the discussion from day to day. Above all, they should try to discover at what points the content of class discussions has been unclear; and they should feel not only free, but actually obligated, to raise questions in class to clear up any confusion. If any individual feels hesitant about asking questions, let him remember that one can hardly raise a question about systematic economic argument which is so simple that most other students will not profit from its discussion.

Students are certain to find this course a more profitable and stimulating intellectual experience if they do their work, at least occasionally, with other students. This is especially true with reference to study of the various problem-exercises. Students can gain a great deal, by way of understanding, if they try to explain things to each other, if they criticize other people’s explanations, and if they attempt to argue out of differences of opinion. It is hard to develop real facility with definitions, concepts, and propositions merely by reading — or by talking to one’s self.

 

Headings from Course Outline
(63 pages)

INTRODUCTION

Definition of Economics and of Its Point of View

Basic Functions or Tasks in an Economic System or Organization

GENERAL PRICE THEORY

[Introduction]

General View of the Pricing Process

The Phenomenon of Industrial Fluctuations and Unemployment, digression

Circularity of the Pricing Process

The Pricing Process: EQUILIBRIUM

The Pricing Process for a Short Period

Conditions of Equilibrium

The Pricing Process over Long Periods

Some Conditions of Long-run Equilibrium

Some Interpretations of the Equilibrium Arrangements

Complexity and Intricacy of the Inter-relations

Some Supplementary Remarks

DEMAND, DEMAND FUNCTIONS, AND ELASTICITY OF DEMAND

Confusion as to Usage of the Word “Demand”

Utility, Utility Functions, and Demand Functions

Elasticity of Demand

COST OF PRODUTION AND PRICE UNDER COMPETITIVE CONDITIONS

Problem Exercise I

Preliminary Exercises
Conditions of Equilibrium in the Industry
Conditions of Demand

[missing pages 40-53]

MONOPOLY AND MONOPOLY PRICE

Contrasts between Complete Monopoly and Perfect Competition

Production and Prices under a special case of Partial Monopoly, “The Economics of Cartels”

An Arithmetic Exercise

 

Source:  University of Chicago Archives. George Stigler Papers,  Addenda, Box 24, Folder “Economics 201”.

Image Source:  Architectural element of the Social Science Research Building (1929). University of Chicago Photographic Archive, apf2-07449, Special Collections Research Center, University of Chicago Library.